You are on page 1of 120

SETS

1.
For an overlapping sets problem it is best to use a double set matrix to organize the
information and solve. Fill in the information in the order in which it is given.
Of the films Empty Set Studios released last year, 60% were comedies and the rest
were horror films.
Comedies
Horror
Films
Total
Profitable
Unprofitable
Total 0.6x 0.4x x
75% of the comedies were profitable, but 75% of the horror moves were
unprofitable.
Comedies
Horror
Films
Total
Profitable 0.75(0.6x
Unprofitable 0.75(0.4x
Total 0.6x 0.4x x
f the studio made a total of !0 films...
Comedies
Horror
Films
Total
Profitable
0.75!4" #
!"
Unprofitable
0.7516"
# !#
Total
0.640" #
#4
0.440" #
!6
x $ 40
$ince each row and each column must sum up to the %otal value& we can fill in the
remaining boxes.
Comedies
Horror
Films
Total
Profitable 1' 4 ##
Unprofitable 6 1! !"
Total !4 16 40
%he problem see(s the total number of profitable films& which is !!.
%he correct answer is ).
!.
For an overlapping sets problem we can use a double*set matrix to organize our
information and solve. +ecause the values are in percents& we can assign a value of
100 for the total number of interns at the hospital. %hen& carefull, fill in the matrix
based on the information provided in the problem. %he matrix below details this
information. -otice that the variable " is used to detail the number of interns who
receive 6 or more hours of sleep& 70. of whom reported no feelings of tiredness.
%ired -ot %ired %/%01
6 or more hours .2" .7" "
Fewer than 6 hours 75 '0
%/%01 100
3n a double*set matrix& the sum of the first two rows e4uals the third and the sum of
the first two columns e4uals the third. %hus& the boldfaced entries below were derived
using the above matrix.
%ired -ot %ired %/%01
6 or more hours 6 !4 #0
Fewer than 6 hours 75 5 '0
%/%01 "! !% 100
5e were as(ed to find the percentage of interns who reported no feelings of tiredness&
or 16. of the interns.
%he correct answer is 7.
2.
%his is an overlapping sets problem concerning two groups students in either band or
orchestra" and the overlap between them students in both band and orchestra".
3f the problem gave information about the students onl, in terms of percents& then a
smart number to use for the total number of students would be 100. 8owever& this
problem gives an actual number of students 9there are 116 students in the band:" in
addition to the percentages given. %herefore& we cannot assume that the total number
of students is 100.
3nstead& first do the problem in terms of percents. %here are three t,pes of students;
those in band& those in orchestra& and those in both. '0. of the students are in onl,
one group. %hus& !0. of the students are in both groups. 50. of the students are in
the band onl,. 5e can use those two figures to determine the percentage of students
left over; 100. * !0. * 50. # 20. of the students are in the orchestra onl,.
<reat * so 20. of the students are in the orchestra onl,. +ut although 20 is an answer
choice& watch out= %he 4uestion doesn>t as( for the percentage of students in the
orchestra onl,& it as(s for the number of students in the orchestra onl,. 5e must
figure out how man, students are in ?usic 8igh $chool altogether.
%he 4uestion tells us that 116 students are in the band. 5e (now that 70. of the
students are in the band; 50. in band onl,& plus !0. in both band and orchestra. 3f
we let x be the total number of students& then 116 students are 70. of x& or 116 # .7x.
%herefore& x # 116 @ .7 # 170 students total.
%he number of students in the orchestra onl, is 20. of 170& or .2 A 170 # 51.
%he correct answer is +.
4.
For an overlapping set problem we can use a double*set matrix to organize our
information and solve. 1et>s call # the number of people at the convention. %he
boldfa&e entries in the matrix below were given in the 4uestion. For example& we are
told that one sixth of the attendees are female students& so we put a value of #@6 in the
female students cell.
F)?01)
-/%
F)?01)
%/%01$
$%BC)-%$ P'6 #@6 P'(
-/-
$%BC)-%$
#@! !50 !#@2
%/%01$ #P'( #@2 P
%he non*boldfaced entries can be derived using simple e4uations that involve the
numbers in one of the DtotalD cells. 1et>s loo( at the DFemaleD column as an example.
$ince we (now the number of female students #@6" and we (now the total number of
females !#@2"& we can set up an e4uation to find the value of female non*students;
#@6 E $emale %on Students # !#@2.
$olving this e4uation ,ields; $emale %on Students # !#@2 F #@6 # #@!.
+, solving the e4uation derived from the D-/% F)?01)D column& we can
determine a value for #.
#
6
E 150
#
#
2


# E 600 # !# #
# 600
%he correct answer is ).
5.
For an overlapping set problem we can use a double*set matrix to organize our
information and solve. +ecause the values here are percents& we can assign a value of
100 to the total number of lights at 8otel 7alifornia. %he information given to us in
the 4uestion is shown in the matrix in boldface. 0n " was assigned to the lights that
were 9$upposed %o +e /ff: since the values given in the problem reference that
amount. %he other values were filled in using the fact that in a double*set matrix the
sum of the first two rows e4uals the third and the sum of the first two columns e4uals
the third.
$upposed
%o +e /n
$upposed
%o +e /ff
%/%01
0ctuall,
on
0.4x "0
0ctuall,
off
0.!(!00 F
x
0.6" !0
%/%01 100 F " x !00
Bsing the relationships inherent in the matrix& we see that;
0.1100 F "" E 0.6" # !0
10 F 0.1" E 0.6" # !0
0.5" # 10 so " # !0
5e can now fill in the matrix with values;
$upposed
%o +e /n
$upposed
%o +e
/ff
%/%01
0ctuall,
on
7! ' '0
0ctuall,
off
' 1! !0
%/%01 '0 !0 100
/f the '0 lights that are actuall, on& '& or 10. percent& are supposed to be off.
%he correct answer is C.
6.
%his 4uestion involves overlapping sets so we can emplo, a double*set matrix to help
us. %he two sets are spec(led@rainbow and male@female. 5e can fill in 645 for the
total number of total spec(led trout based on the first sentence. 0lso& we can assign a
variable& "& for female spec(led trout and the expression !" E 45 for male spec(led
trout& also based on the first sentence.
?ale Female %otal
$pec(led !" E 45 " 645
Gainbow
%otal
5e can solve for " with the following e4uation; 2" E 45 # 645. %herefore& " # !00.
?ale Female %otal
$pec(led 445 !00 645
Gainbow
%otal
3f the ratio of female spec(led trout to male rainbow trout is 4;2& then there must be
150 male rainbow trout. 5e can easil, solve for this with the below proportion where
y represents male rainbow trout;
%herefore& y # 150. 0lso& if the ratio of male rainbow trout to all trout is 2;!0& then
there must be 1000 total trout using the below proportion& where & represents all trout;
4
2
#
!00
y
2
!0
#
150
&
?ale Female %otal
$pec(led 445
!00
645
Gainbow 150
%otal 1000
-ow we can Hust fill in the empt, boxes to get the number of female rainbow trout.
?ale Female %otal
$pec(led 445
!00 645
Gainbow
150
!05 255
%otal 1000
%he correct answer is C.
7.
+egin b, constructing a double*set matrix and filling in the information given in
the problem. 0ssume there are 100 maHor airline companies in total since this is an
eas, number to wor( with when dealing with percent problems.
5ireless -o
5ireless
%/%01
$nac(s
)
*+, )
70
-/
$nac(s
20
%/%01 20 70 100
-otice that we are tr,ing to maximize the cell where wireless intersects with snac's.
5hat is the maximum possible value we could put in this cell. $ince the total of the
snac(s row is 70 and the total of the wireless column is 20& it is clear that 20 is the
limiting number. %he maximum value we can put in the wireless*snac(s cell is
therefore 20. 5e can put 20 in this cell and then complete the rest of the matrix to
ensure that all the sums will wor( correctl,.
5ireless -o
5ireless
%/%01
$nac(s (0 40 70
-/
$nac(s
0 20 20
%/%01 20 70 100
%he correct answer is +.
'.
For an overlapping set problem we can use a double*set matrix to organize our
information and solve. +ecause the given values are all percentages& we can assign a
value of 100 to the total number of people in countr, I. %he matrix is filled out below
based on the information provided in the 4uestion.
%he first sentence tells us that 10. of all of the people do have their Hob of choice but
do not have a diploma& so we can enter a 10 into the relevant box& below. %he second
sentence tells us that !5. of those who do not have their (ob of choice have a
diploma. 5e don>t (now how man, people do not have their Hob of choice& so we
enter a variable in this case& "" into that box. -ow we can enter !5. of those people&
or 0.!5x& into the relevant box& below. Finall,& we>re told that 40. of all of the
people have their Hob of choice.
Bniversit, Ciploma -/ Bniversit, Ciploma %/%01
Job of 7hoice 10 40
-/% Job of 7hoice 0.!5" "
%/%01 100
3n a double*set matrix& the sum of the first two rows e4uals the third and the sum of
the first two columns e4uals the third. %hus& the boldfaced entries below were derived
using relationships for example; 40 E " # 100& therefore " # 60. 0.!5 A 60 # 15. 0nd
so on.".
Bniversit, Ciploma -/ Bniversit, Ciploma
%/%01
Job of 7hoice
(0 10 40
-/% Job of 7hoice !5 45 60
%/%01 45 55 100
5e were as(ed to find the percent of the people who have a universit, diploma& or
45..
%he correct answer is +.
6.
%his is a problem that involves two overlapping sets so it can be solved using a
double*set matrix. %he problem tells us that there are '00 total students of whom 70.
or 560 are male. %his means that !40 are female and we can begin filling in the
matrix as follows;
?ale Female %/%01
$port
-o $port ma"imi&e
%/%01 560 !40 '00
%he 4uestion as(s us to ?0K3?3I) the total number of students who do -/%
participate in a sport. 3n order to maximize this total& we will need to maximize the
number of females who do -/% participate in and the number of males who do -/%
participate in a sport.
%he problem states that at least 10. of the female students& or !4 female students&
participate in a sport. %his leaves !16 female students who ma, or ma, not participate
in a sport. $ince we want to maximize the number of female students who do -/%
participate in a sport& we will assume that all !16 of these remaining female students
do not participate in a sport.
%he problem states that fewer than 20. of the male students do -/% participate in a
sport. %hus& fewer than 16' male students 20. of 560" do -/% participate in a
sport. %hus an,where from 0 to 167 male students do -/% participate in a sport.
$ince we want to maximize the number of male students who do -/% participate in a
sport& we will assume that 167 male students do -/% participate in a sport. %his
leaves 262 male students who do participate in a sport.
%hus& our matrix can now be completed as follows;
?ale Female %/%01
$port 262 !4 417
-o $port 167 !16 ("(
%/%01 560 !40 '00
%herefore& the maximum possible number of students in $chool % who do not
participate in a sport is 2'2.
%he correct answer is +.
!0.
%his is an overlapping sets problem& which can be solved most efficientl, b, using a
double set matrix. /ur first step in using the double set matrix is to fill in the
information given in the 4uestion. +ecause there are no real values given in the
4uestion& the problem can be solved more easil, using >smart numbers>L in this case&
we can assume the total number of rooms to be 100 since we are dealing with
percentages. 5ith this assumption& we can fill the following information into our
matrix;
%here are 100 rooms total at the $tagecoach 3nn.
/f those 100 rooms& 75 have a 4ueen*sized bed& while !5 have a (ing*sized bed.
/f the non*smo(ing rooms let>s call this un(nown n"& 60. or .6n have 4ueen*sized
beds.
10 rooms are non*smo(ing with (ing*sized beds.
1et>s fill this information into the double set matrix& including the variable n for the
value we need to solve the problem;
$?/M3-< -/-*$?/M3-< %/%01$
M3-< +)C 10 !5
NB))- +)C .6n 75
%/%01$ n 100
3n a double*set matrix& the first two rows sum to the third& and the first two columns
sum to the third. 5e can therefore solve for n using basic algebra;
10 E .6n # n
10 # .4n
n # !5
5e could solve for the remaining empt, fields& but this is unnecessar, wor(.
/bserve that the total number of smo(ing rooms e4uals 100 F n # 100 F !5 # 75.
Gecall that we are wor(ing with smart numbers that represent percentages& so 75. of
the rooms at the $tagecoach 3nn permit smo(ing.
%he correct answer is ).
!!.
For an overlapping set problem we can use a double*set matrix to organize our
information and solve. %he boldfaced values were given in the 4uestion. %he non*
boldfaced values were derived using the fact that in a double*set matrix& the sum of
the first two rows e4uals the third and the sum of the first two columns e4uals the
third. %he variable p was used for the total number of pin( roses& so that the total
number of pin( and red roses could be solved using the additional information given
in the 4uestion.

Ged Oin( 5hite %/%01
1ong*
stemmed
0 '0
$hort*
stemmed
5 !5 #0 40
%/%01 100 * p p !0 !#0
%he 4uestion states that the percentage of red roses that are short*stemmed is e4ual to
the percentage of pin( roses that are short stemmed& so we can set up the following
proportion;
5p # 1500 F 15p
p # 75
%his means that there are a total of 75 pin( roses and !5 red roses. -ow we can fill
out the rest of the double*set matrix;
Ged Oin( 5hite %/%01
1ong*
stemmed
!0 60 0 '0
$hort*
stemmed
5 !5 #0 40
%/%01 !5 75 !0 !#0
-ow we can answer the 4uestion. !0 of the '0 long*stemmed roses are red& or !0@'0
# !5..
%he correct answer is +.
!#.
%he best wa, to approach this 4uestion is to construct a matrix for each town. 1et>s
start with %own K. $ince we are not given an, values& we will insert un(nowns into
the matrix;
1eft*8anded -ot 1eft*8anded %otal
%all 0 7 0 E 7
-ot %all + C + E C
%otal 0 E + 7 E C 0 E + E 7 E C
-ow let>s create a matrix for %own P& using the information from the 4uestion and
the un(nowns from the matrix for %own K;
1eft*8anded -ot 1eft*8anded %otal
%all 20 27 20 E 27
5
100 F p
#
15
p
-ot %all 2+ 0 2+
%otal 20 E 2+ 27 20 E 2+ E27
$ince we (now that the total number of people in %own K is four times greater than
the total number of people in %own P& we can construct and simplif, the following
e4uation;
$ince ) represents the number of people in %own K who are neither tall nor left*
handed& we (now that the correct answer must be a multiple of 11. %he onl, answer
choice that is a multiple of 11 is 142 .
%he correct answer is C.
!(.
Pou can solve this problem with a matrix. $ince the total number of diners is
un(nown and not important in solving the problem& wor( with a h,pothetical total of
100 couples. $ince ,ou are dealing with percentages& 100 will ma(e the math easier.
$et up the matrix as shown below;
-essert
./
dessert
T/T+0
Coffee
./ &offee
T/T+0 !00
$ince ,ou (now that 60. of the couples order +/%8 dessert and coffee& ,ou can
enter that number into the matrix in the upper left cell.
-essert
./
dessert
T/T+0
Coffee 60
./ &offee
T/T+0 100
%he next useful piece of information is that !0. of the couples who order dessert
don>t order coffee. 12t be &aref2l3 %he problem does not sa, that !0. of the total
diners order dessert and don>t order coffee& so ,ou 70--/% fill in 40 under
Ddessert& no coffeeD first column& middle row". 3nstead& ,ou are told that !0. of t4e
&o2ples 54o order dessert don>t order coffee.
1et x # total number of couples who order dessert. %herefore ,ou can fill in .2x for
the number of couples who order dessert but no coffee.
-essert
./
dessert
T/T+0
Coffee 60
./ &offee .2x
T/T+0 x 100
$et up an e4uation to represent the couples that order dessert and solve;
75. of all couples order dessert. %herefore& there is onl, a !5. chance that the next
couple the maitre >d seats will not order dessert. %he correct answer is +.
!4.
%his problem involves two sets;
$et 1; 0partments with windows @ 0partments without windows
$et !; 0partments with hardwood floors @ 0partments without hardwood floors.
3t is easiest to organize two*set problems b, using a matrix as follows;
6indo5s ./ 6indo5s T/T+0
Hard5ood Floors
./ Hard5ood
Floors
T/T+0
%he problem is difficult for two reasons. First& it uses percents instead of real
numbers. $econd& it involves complicated and subtle wording.
1et>s attac( the first difficult, b, converting all of the percentages into G)01
numbers. %o do this& let>s sa, that there are 100 total apartments in the building. %his
is the first number we can put into our matrix. %he absolute total is placed in the
lower right hand corner of the matrix as follows;
6indo5s ./ 6indo5s T/T+0
Hard5ood Floors
./ Hard5ood
Floors
T/T+0 !00
-ext& we will attac( the complex wording b, reading each piece of information
separatel,& and filling in the matrix accordingl,.
3nformation; 507 of t4e apartments in a &ertain b2ildin8 4a9e 5indo5s and
4ard5ood floors. %hus& 50 of the 100 apartments have +/%8 windows and
hardwood floors. %his number is now added to the matrix;
6indo5s ./ 6indo5s T/T+0
Hard5ood Floors 50
./ Hard5ood
Floors
T/T+0 !00
3nformation; #57 of t4e apartments 5it4o2t 5indo5s 4a9e 4ard5ood floors.
8ere>s where the subtlet, of the wording is ver, important. %his does -/% sa, that
!5. of 011 the apartments have no windows and have hardwood floors. 3nstead it
sa,s that /F the apartments without windows& !5. have hardwood floors. $ince we
do not ,et (now the number of apartments without windows& let>s call this number x.
%hus the number of apartments without windows and with hardwood floors is .#5x.
%hese figures are now added to the matrix;
5indows -/ 5indows %/%01
8ardwood Floors 50 .#5x
-/ 8ardwood
Floors
%/%01 x !00
3nformation; 407 of t4e apartments do not 4a9e 4ard5ood floors. %hus& 40 of the
100 apartments do not have hardwood floors. %his number is put in the %otal box at
the end of the D-o 8ardwood FloorsD row of the matrix;
5indows -/ 5indows %/%01
8ardwood Floors 50 .#5x
-/ 8ardwood
Floors
40
%/%01 x
+efore answering the 4uestion& we must complete the matrix. %o do this& fill in the
numbers that ,ield the given totals. First& we see that there must be be 60 total
apartments with 8ardwood Floors since 60 E 40 # 100" Bsing this information& we
can solve for x b, creating an e4uation for the first row of the matrix;
-ow we put these numbers in the matrix;
5indows -/ 5indows %/%01
8ardwood Floors 50 !0
60
-/ 8ardwood
Floors
40
%/%01 40 !00
Finall,& we can fill in the rest of the matrix;
5indows -/ 5indows %/%01
8ardwood Floors 50 !0
60
-/ 8ardwood
Floors !0 (0
40
%/%01
60
40 !00
5e now return to the 4uestion; 5hat percent of the apartments with windows have
hardwood floorsQ
0gain& pa, ver, careful attention to the subtle wording. %he 4uestion does -/% as(
for the percentage of %/%01 apartments that have windows and hardwood floors. 3t
as(s what percent /F the apartments with windows have hardwood floors. $ince
there are 60 apartments with windows& and 50 of these have hardwood floors& the
percentage is calculated as follows;
%hus& the correct answer is ).
15.
%his problem can be solved using a set of three e4uations with three un(nowns. 5e>ll
use the following definitions;
1et F # the number of FuHi trees
1et < # the number of <ala trees
1et 7 # the number of cross pollinated trees
10. of his trees cross pollinated
7 # 0.1F E < E 7"
107 # F E < E 7
67 # F E <
%he pure FuHis plus the cross pollinated ones total 1'7
4" F E 7 # 1'7
2@4 of his trees are pure FuHi
5" F # 2@4"F E < E 7"
6" 4F # 2F E 2< E 27
7" F # 2< E 27
$ubstituting the value of F from e4uation 7" into e4uation 2" gives us;
'" 67 # 2< E 27" E <
6" 67 # 4<
10" 1!7 # '<
$ubstituting the value of F from e4uation 7" into e4uation 4" gives us;
11" 2< E 27" E 7 # 1'7
1!" 2< E 47 # 1'7
12" 6< E 1!7 # 561
$ubstituting e4uation 10" into 12" gives;
14" 6< E '< # 561
15" 17< # 561
16" < # 22
$o the farmer has 22 trees that are pure <ala.
%he correct answer is +.
16.
For an overlapping set problem with three subsets& we can use a Renn diagram to
solve.
)ach circle represents the number of students enrolled in the 8istor,& )nglish and
?ath classes& respectivel,. -otice that each circle is subdivided into different groups
of students. <roups a& e& and f are comprised of students ta(ing onl, 1 class. <roups
b& c, and d are comprised of students ta(ing ! classes. 3n addition& the diagram shows
us that 2 students are ta(ing all 2 classes. 5e can use the diagram and the information
in the 4uestion to write several e4uations;
8istor, students; a E b E c E 2 # !5
?ath students; e E b E d E 2 # !5
)nglish students; f E c E d E 2 # 24
%/%01 students; a E e E f E b E c E d E 2 # 6'
%he 4uestion as(s for the total number of students ta(ing exactl, ! classes. %his can
be represented as b E c E d.
3f we sum the first 2 e4uations 8istor,& ?ath and )nglish" we get;
a E e E f E !b E!c E!d E 6 # '4.
%a(ing this e4uation and subtracting the 4
th
e4uation %otal students" ,ields the
following;
a * e * f * !b E !c E!d * 6 # '4
FS a * e * f * b * c * d * 2 # 6'T
b * c * d # 10
%he correct answer is +.
17. %his is a three*set overlapping sets problem. 5hen given three sets& a Renn
diagram can be used. %he first step in constructing a Renn diagram is to identif,
the three sets given. 3n this case& we have students signing up for the poetr, club&
the histor, club& and the writing club. %he shell of the Renn diagram will loo(
li(e this;
5hen filling in the regions of a Renn diagram& it is important to wor( from inside
out. 3f we let " represent the number of students who sign up for all three clubs& a
represent the number of students who sign up for poetr, and writing& b represent
the number of students who sign up for poetr, and histor,& and c represent the
number of students who sign up for histor, and writing& the Renn diagram will
loo( li(e this;
5e are told that the total number of poetr, club members is !!& the total number
of histor, club members is !7& and the total number of writing club members is
!'. 5e can use this information to fill in the rest of the diagram;
5e can now derive an expression for the total number of students b, adding up
all the individual segments of the diagram. %he first brac(eted item represents the
students ta(ing two or three courses. %he second brac(eted item represents the
number of students in onl, the poetr, club& since it>s derived b, adding in the total
number of poetr, students and subtracting out the poetr, students in multiple
clubs. %he third and fourth brac(eted items represent the students in onl, the
histor, or writing clubs respectivel,.
56 # Sa E b E c E "T E S!! F a E b E ""T E S!7 F b E c E ""T E S!' F a E c E ""T
56 # a E b E c E " E !! F a F b F " E !7 F b F c F " E !' F a F c F "
56 # 77 F !" F a F b F c
56 # 77 F !" F a E b E c"
+, examining the diagram& we can see that a E b E c" represents the total number
of students who sign up for two clubs. 5e are told that 6 students sign up for
exactl, two clubs. 7onse4uentl,;
56 # 77 F !" F 6
!" # 1!
" # 6
$o& the number of students who sign up for all three clubs is 6.
0lternativel,& we can use a more intuitive approach to solve this problem. 3f we
add up the total number of club sign*ups& or registrations& we get !! E !7 E !' #
77. 5e must remember that this number includes overlapping registrations some
students sign up for two clubs& others for three". $o& there are 77 registrations and
56 total students. %herefore& there must be 77 F 56 # 1' duplicate registrations.
5e (now that 6 of these duplicates come from those 6 students who sign up for
exactl, two clubs. )ach of these 6& then& adds one extra registration& for a total of
6 duplicates. 5e are then left with 1' F 6 # 1! duplicate registrations. %hese 1!
duplicates must come from those students who sign up for all three clubs.
For each student who signs up for three clubs& there are two extra sign*ups.
%herefore& there must be 6 students who sign up for three clubs;
1! duplicates @ ! duplicates@student" # 6 students
+etween the 6 students who sign up for two clubs and the 6 students who sign up
for all three& we have accounted for all 1' duplicate registrations.
%he correct answer is 7.
!".
%his problem involves 2 overlapping sets. %o visualize a 2 set problem& it is best to
draw a Renn Ciagram.
5e can begin filling in our Renn Ciagram utilizing the following ! facts; 1" %he
number of bags that contain onl, raisins is 10 times the number of bags that contain
onl, peanuts. !" %he number of bags that contain onl, almonds is !0 times the
number of bags that contain onl, raisins and peanuts.
-ext& we are told that the number of bags that contain onl, peanuts which we have
represented as x" is one*fifth the number of bags that contain onl, almonds which we
have represented as !0,".
%his ,ields the following e4uation; x # 1@5"!0, which simplifies to x # 4,. 5e can
use this information to revise our Renn Ciagram b, substituting an, x in our original
diagram with 4, as follows;
-otice that& in addition to performing this substitution& we have also filled in the
remaining open spaces in the diagram with the variable a& b& and c.
-ow we can use the numbers given in the problem to write ! e4uations. First& the sum
of all the expressions in the diagram e4uals 425 since we are told that there are 425
bags in total. $econd& the sum of all the expressions in the almonds circle e4uals !10
since we are told that !10 bags contain almonds.
425 # !0, E a E b E c E 40, E , E 4,
!10 # !0, E a E b E c
$ubtracting the second e4uation from the first e4uation& ,ields the following;
!!5 # 40, E , E 4,
!!5 # 45,
5 # ,
<iven that , # 5& we can determine the number of bags that contain onl, one (ind of
item as follows;
%he number of bags that contain onl, raisins # 40, # !00
%he number of bags that contain onl, almonds # !0, # 100
%he number of bags that contain onl, peanuts # 4, # !0
%hus there are 2!0 bags that contain onl, one (ind of item. %he correct answer is C.
16.
%his is an overlapping sets problem. %his 4uestion can be effectivel, solved with a
double*set matrix composed of two overlapping sets; S$panish@-ot $panishT and
SFrench@-ot FrenchT. 5hen constructing a double*set matrix& remember that the two
categories adHacent to each other must be mutuall, exclusive& i.e. SFrench@not FrenchT
are mutuall, exclusive& but SFrench@not $panishT are not mutuall,
exclusive. Following these rules& letUs construct and fill in a double*set matrix for
each statement. %o simplif, our wor( with percentages& we will also pic( 100 for the
total number of students at Jefferson 8igh $chool.
3-$BFF373)-%; 5hile we (now the percentage of students who ta(e French
and& from that information& the percentage of students who do not ta(e French&
we do not (now an,thing about the students ta(ing $panish. %herefore we
don>t (now the percentage of students who stud, French but not $panish& i.e.
the number in the target cell denoted with ".
FG)-78 -/% FG)-78 %/%01$
$O0-3$8
-/% $O0-3$8 "
%/%01$ 20 70 100
!" 3-$BFF373)-%; 5hile we (now the percentage of students who do not ta(e
$panish and& from that information& the percentage of students who do ta(e $panish&
we do not (now an,thing about the students ta(ing French. %herefore we don>t (now
the percentage of students who stud, French but not $panish& i.e. the number in the
target cell denoted with ".
FG)-78 -/% FG)-78 %/%01$
$O0-3$8 60
-/% $O0-3$8 " 40
%/%01$ 100
0-C !" 3-$BFF373)-%; )ven after we combine the two statements& we do
not have sufficient information to find the percentage of students who stud,
French but not $panish& i.e. to fill in the target cell denoted with ".
FG)-78 -/% FG)-78 %/%01$
$O0-3$8 60
-/% $O0-3$8 " 40
%/%01$ 20 70 100
%he correct answer is ).
!0.
For this overlapping sets problem& we want to set up a double*set matrix. %he first set
is bo,s vs. girlsL the second set is left*handers vs. right*handers.
%he onl, number currentl, in our chart is that given in the 4uestion; !0& the total
number of students.
<3G1$ +/P$ %/%01$
1)F%*80-C)C
G3<8%*80-C)C
%/%01$ !0
3-$BFF373)-%; 5e can figure out that three girls are left*handed& but we
(now nothing about the bo,s.
<3G1$ +/P$ %/%01$
1)F%*80-C)C 0.!5"1!" # 2
G3<8%*80-C)C
%/%01$ 1! !0
!" 3-$BFF373)-%; 5e can>t figure out the number of left*handed bo,s& and we
(now nothing about the girls.
<3G1$ +/P$ %/%01$
1)F%*80-C)C
G3<8%*80-C)C 5
%/%01$ !0
1" 0-C !" $BFF373)-%; 3f we combine both statements& we can get the missing
pieces we need to solve the problem. $ince we have 1! girls& we (now that there are '
bo,s. 3f five of them are right*handed& then three of them must be left*handed. 0dd
that to the three left*handed girls& and we (now that a total of 6 students are left*
handed.
<3G1$ +/P$ %/%01$
1)F%*80-C)C 2 ( 6
G3<8%*80-C)C 5
%/%01$ 1! " !0
%he correct answer is 7.
!1.
For this overlapping set problem& we want to set up a two*set table to test our
possibilities. /ur first set is vegetarians vs. non*vegetariansL our second set is
students vs. non*students.
R)<)%0G30- -/-*R)<)%0G30- %/%01
$%BC)-%
-/-*$%BC)-% 15
%/%01 " " Q
5e are told that each non*vegetarian non*student ate exactl, one of the 15
hamburgers& and that nobod, else ate an, of the 15 hamburgers. %his means that there
were exactl, 15 people in the non*vegetarian non*student categor,. 5e are also told
that the total number of vegetarians was e4ual to the total number of non*vegetariansL
we represent this b, putting the same variable in both boxes of the chart.
%he 4uestion is as(ing us how man, people attended the part,L in other words& we are
being as(ed for the number that belongs in the bottom*right box& where we have
placed a 4uestion mar(.
%he second statement is easier than the first statement& so we>ll start with statement
!".
!" 3-$BFF373)-%; %his statement gives us information onl, about the cell labeled
Dvegetarian non*studentDL further it onl, tells us the number of these guests as a
percenta+e of the total guests. %he 20. figure does not allow us to calculate the
actual number of an, of the categories.
$BFF373)-%; %his statement provides two pieces of information. First& the
vegetarians attended at the rate& or in the ratio& of !;2 students to non*
students. 5e>re also told that this !;2 rate is half the rate for non*vegetarians.
3n order to double a rate& we double the first numberL the rate for non*
vegetarians is 4;2 5e can represent the actual numbers of non*vegetarians as
4a and 2a and add this to the chart below. $ince we (now that there were 15
non*vegetarian non*students& we (now the missing common multiple& a& is
15@2 # 5. %herefore& there were 4"5" # !0 non*vegetarian students and !0 E
15 # 25 total non*vegetarians see the chart below". $ince the same number of
vegetarians and non*vegetarians attended the part,& there were also 25
vegetarians& for a total of 70 guests.
R)<)%0G30- -/-*R)<)%0G30- %/%01
$%BC)-% 4a or !0
-/-*$%BC)-% 2a or 15
%/%01 " or 25 " or 25 Q or 70
%he correct answer is 0.
!!.
For an overlapping set 4uestion& we can use a double*set matrix to organize the
information and solve. %he two sets in this 4uestion are the practical test pass@fail"
and the written test pass@fail".
From the 4uestion we can fill in the matrix as follows. 3n a double*set matrix& the sum
of the first two rows e4uals the third and the sum of the first two columns e4uals the
third. %he bolded value was derived from the other given values. %he 4uestion as(s us
to find the value of .7"
OG07%3701 * O0$$ OG07%3701 * F031 %/%01$
5G3%%)- * O0$$
.7x .2" "
5G3%%)- * F031
0
%/%01$
.2"
1" 3-$BFF373)-%; 3f we add the total number of students to the information from
the 4uestion& we do not have enough to solve for .7".
OG07%3701 * O0$$ OG07%3701 * F031 %/%01$
5G3%%)- * O0$$
.7x .2" "
5G3%%)- * F031
0
%/%01$
.2"
1''
!" 3-$BFF373)-%; 3f we add the fact that !0. of the si"teen year,olds who passed
the practical test failed the written test to the original matrix from the 4uestion& we
can come up with the relationship .7" # .'y. 8owever& that is not enough to solve for .
7".
OG07%3701 * O0$$ OG07%3701 * F031 %/%01$
5G3%%)- * O0$$
.7x = ."y .2" "
5G3%%)- * F031
.!y 0 .!y
%/%01$
y .2"
1" 0-C !" $BFF373)-%; 3f we combine the two statements we get a matrix that
can be used to form two relationships between " and y;
OG07%3701 * O0$$ OG07%3701 * F031 %/%01$
5G3%%)- * O0$$
.7x = ."y .2" "
5G3%%)- * F031
.!y 0 .!y
%/%01$
y .2"
1''
.7" # .'y
y E .2" # 1''
%his would allow us to solve for " and in turn find the value of .7"& the number of
sixteen ,ear*olds who received a driver license.
%he correct answer is 7.
!2.
For an overlapping set problem we can use a double*set matrix to organize our
information and solve. 5e are told in the 4uestion stem that 1'0 guests have a house
in the 8amptons and a house in Oalm +each. 5e can insert this into our matrix as
follows;
8ouse in
8amptons
-o 8ouse
in
8amptons
%/%01$
8ouse in
Oalm
+each
1'0
-o
8ouse in
Oalm
+each
%/%01$ -
%he 4uestion is as(ing us for the ratio of the dar(l, shaded box to the lightl, shaded
box.
3-$BFF373)-%; $ince one*half of all the guests had a house in Oalm +each&
we can fill in the matrix as follows;
8ouse in
8amptons
-o 8ouse
in
8amptons
%/%01$
8ouse in
Oalm
+each
1'0
1@!"- F
1'0
1@!"-
-o
8ouse in
Oalm
+each
%/%01$ -
5e cannot find the ratio of the dar( box to the light box from this information alone.
!" 3-$BFF373)-%; $tatement ! tells us that two*thirds of all the guests had a house
in the 8amptons. 5e can insert this into our matrix as follows;
8ouse in
8amptons
-o 8ouse
in %/%01$
8amptons
8ouse in
Oalm
+each
1'0
-o
8ouse in
Oalm
+each
!@2"- F
1'0
%/%01$ !@2"- -
5e cannot find the ratio of the dar( box to the light box from this information alone.
1" 0-C !" 3-$BFF373)-%; we can fill in our matrix as follows.
8ouse in
8amptons
-o 8ouse
in
8amptons
%/%01$
8ouse in
Oalm
+each
1'0
1@!"- F
1'0 1@!"-
-o
8ouse in
Oalm
+each
!@2"- F
1'0
!"0 :
(!'6T
(!'#T
%/%01$ !@2"- (!'(T -
%he ratio of the number of people who had a house in Oalm +each but not in the
8amptons to the number of people who had a house in the 8amptons but not in Oalm
+each i.e. dar( to light" will be;
%his ratio doesnUt have a constant valueL it depends on the value of -. 5e can tr, to
solve for - b, filling out the rest of the values in the matrix see the bold entries
above"L however& an, e4uation that we would build using these values reduces to a
redundant statement of - # -. %his means there isnUt enough uni4ue information to
solve for %.
1@!"-
,1'0
!@2"- *
1'0
%he correct answer is ).
!4.
$ince there are two different classes into which we can divide the participants& we can
solve this using a double*set matrix. %he two classes into which we>ll divide the
participants are +o,s@<irls along the top as column labels"& and
7hocolate@$trawberr, down the left as row labels".
%he problem gives us the following data to fill in the initial double*set matrix. 5e
want to (now if we can determine the maximum value of a& which represents the
number of girls who ate chocolate ice cream.
+/P$ <3G1$ %/%01$
78/7/10%) ' a
$%G05+)GGP 6
%/%01$
1" $BFF373)-%; $tatement 1" tells us that exactl, 20 children came to the part,&
so we>ll fill in 20 for the grand total. Gemember that we>re tr,ing to maximize a.
+/P$ <3G1$ %/%01$
78/7/10%) ' a b
$%G05+)GGP c 6 d
%/%01$ 20
3n order to maximize a& we must maximize b& the total number of chocolate eaters.
$ince
b E d # 20& impl,ing b # 20 * d& we must minimize d to maximize b. %o minimize d
we must minimize c. %he minimum value for c is 0& since the 4uestion doesn>t sa,
that there were necessaril, bo,s who had strawberr, ice cream.
-ow that we have an actual value for c& we can calculate forward to get the maximum
possible value for a. 3f c # 0& since we (now that c E 6 # d& then d # 6. $ince b E d #
20& then b # !1. <iven that ' E a # b and b # !1& then a # 12& the maximum value we
were loo(ing for. %herefore statement 1" is sufficient to find the maximum number
of girls who ate chocolate.
!" 3-$BFF373)-%; Mnowing onl, that fewer than half of the people ate strawberr,
ice cream doesn>t allow us to fill in an, of the boxes with an, concrete numbers.
%herefore statement !" is insufficient.
%he correct answer is 0.
#5.
$ince we are dealing with overlapping sets and there are two independent criteria& the
best wa, to solve this problem is with a double*set matrix.
%he first statement in the 4uestion stem tells us that of the students who spea( French
represented b, the first column"& four times as man, spea( <erman as don>t. %his
information ,ields the following entries in the double*set matrix;
FG)-78 -/ FG)-78 %/%01$
<)G?0- 4"
-/ <)G?0- "
%/%01$
%he second statement in the 4uestion stem tells us that 1@6 of the of the students who
don>t spea( <erman do spea( French. %his is fact represented in the double*set matrix
as follows;
FG)-78 -/ FG)-78 %/%01$
<)G?0- 4"
-/ <)G?0- " . y@6 y
%/%01$
-ow since " # y@6& we can get rid of the new variable y and (eep all the expressions in
terms of ".
FG)-78 -/ FG)-78 %/%01$
<)G?0- 4"
-/ <)G?0- " 6"
%/%01$
-ow we can fill in a few more boxes using the addition rules for the double*set
matrix.
FG)-78 -/ FG)-78 %/%01$
<)G?0- 4"
-/ <)G?0- " 5" 6"
%/%01$ 5"
%he main 4uestion to be answered is what fraction of the students spea( <erman& a
fraction represented b, /@0 in the final double*set matrix. $o& if statements 1" and@or
!" allow us to calculate a numerical value for /@0, we will be able to answer the
4uestion.
FG)-78 -/ FG)-78 %/%01$
<)G?0- !" /
-/ <)G?0- " 5" 6"
%/%01$ 5" 0
1" 3-$BFF373)-%; $tatement 1" tells us that 60 students spea( French and
<erman& so 4" # 60 and " # 15. 5e can now calculate an, box labeled with an "& but
this is still insufficient to calculate /& 0& or /@0.
!" 3-$BFF373)-%; $tatement !" tells us that 75 students spea( neither French nor
<erman& so 5" # 75 and " # 15. Just as with $tatement 1"& we can now calculate an,
box labeled with an "& but this is still insufficient to calculate /& 0& or /@0.
1" 0-C !" 3-$BFF373)-%; $ince both statements give us the same information
namel,& that " # 15"& putting the two statements together does not tell us an,thing
new. %herefore 1" and !" together are insufficient.
%he correct answer is ).
#6.
3n an overlapping set problem& we can use a double set matrix to organize the
information and solve.
From information given in the 4uestion& we can fill in the matrix as follows;
<G)P 583%) %/%01$
+1B) V 2
+G/5-
%/%01$ 55
%he 4uestion is as(ing us if the total number of blue*e,ed wolves fourth column&
second row" is greater than the total number of brown*e,ed wolves fourth column&
third row".
1" 3-$BFF373)-%. %his statement allows us to fill in the matrix as below. 5e have
no information about the total number of brown*e,ed wolves.
<G)P 583%) %/%01$
+1B) 4" 2" 7x
+G/5-
%/%01$ 55
!" 3-$BFF373)-%. %his statement allows us to fill in the matrix as below. 5e have
no information about the total number of blue*e,ed wolves.
<G)P 583%) %/%01$
+1B)
+G/5- 1y !y (y
%/%01$ 55
%/<)%8)G& statements 1" E !" are $BFF373)-%. 7ombining both statements& we
can fill in the matrix as follows;
<G)P 583%) %/%01$
+1B) 4" 2" 7x
+G/5- 1y !y (y
%/%01$ 4x ; y (x ; #y 55
Bsing the additive relationships in the matrix& we can derive the e4uation 7" E 2y #
55 notice that adding the gre, and white totals ,ields the same e4uation as adding the
blue and brown totals".
%he original 4uestion can be rephrased as D3s 7" V 2y1D
/n the surface& there seems to -/% be enough information to solve this
4uestion. 8owever& we must consider some of the restrictions that are placed on the
values of " and y2
1" x and y m2st be inte8ers we are tal(ing about numbers of wolves here and
loo(ing at the table& y& 2" and 4" must be integers so " and y must be integers"
!" x m2st be 8reater t4an ! the problem sa,s there are more than 2 blue*e,ed
wolves with white coats so 2x must be greater than 2 or x V 1"
$ince " and y must be integers& there are onl, a few "&y values that satisf, the
e4uation 7" E 2y # 55. +, tr,ing all integer values for x from 1 to 7& we can see that
the onl, possible x&, pairs are;
x , 7x 2,
1 16 7 4'
4 % #" #7
7 # 4% 6
$ince " cannot be 1& the onl, two pairs ,ield 7" values that are greater than the
corresponding 2y values !' V !7 and 46 V 6".
%he correct answer is 7.
!7.
5e can divide the current fourth graders into 4 categories;
%he percentage that dressed in costume this ,ear /-1P.
!" %he percentage that dressed in costume last ,ear /-1P.
2" %he percentage that did -/% dress in costume either this ,ear or last ,ear.
4" %he percentage that dressed in costume +/%8 ,ears.
5e need to determine the last categor, categor, 4" in order to answer the
4uestion.
3-$BFF373)-%; 1et>s assume there are 100 current fourth graders this
simpl, helps to ma(e this percentage 4uestion more concrete". 60 of them
dressed in costume this ,ear& while 40 did not. 8owever& we don>t (now how
man, of these 60 dressed in costume last ,ear& so we can>t divide this 60 up
into categories 1 and !.
3-$BFF373)-%; %his provides little relevant information on its own because
we don>t (now how man, of the students didn>t dress up in costumes this ,ear
and the statement references that value.
1" 0-C !" 3-$BFF373)-%; From statement 1 we (now that 60 dressed up
in costumes this ,ear& but 40 did not. $tatement ! tells us that '0. of these
40& or 2!& didn>t dress up in costumes this ,ear either. %his provides us with a
value for categor, 2& from which we can derive a value for categor, ! '".
8owever& we still don>t (now how man, of the 60 costume bearers from this
,ear wore costumes last ,ear.
$ince this is an overlapping set problem& we could also have used a double*set
matrix to organize our information and solve. )ven with both statements
together& we can not find the value for the 7ostume 1ast Pear @ 7ostume %his
Pear cell.
7ostume %his Pear -o 7ostume %his Pear %/%01$
7ostume 1ast Pear '
-o 7ostume 1ast Pear 2!
%/%01$ 60 40 100
%he correct answer is ).
!'.
0 Renn*Ciagram is useful to visualize this problem.
-otice that the Renn diagram allows us to see the 7 different t,pes of houses on
Mermit lane. )ach part of the diagram represents one t,pe of house. For example& the
center section of the diagram represents the houses that contain all three amenities
front ,ard& front porch& and bac( ,ard". Meep in mind that there ma, also be some
houses on Mermit 1ane that have none of the 2 amenities and so these houses would
be outside the diagram.
$BFF373)-%; %his tells us that no house on Mermit 1ane is without a
bac(,ard. )ssentiall, this means that there are 0 houses in the three sections
of the diagram that are -/% contained in the +ac( Pard circle. 3t also means
that there are 0 houses outside of the diagram. $ince we (now that 40 houses
on Mermit 1ane contain a bac( ,ard& there must be exactl, 40 houses on
Mermit 1ane.
3-$BFF373)-%; %his tells us that each house on Mermit 1ane that has a
front porch does not have a front ,ard. %his means that there are 0 houses in
the two sections of the diagram in which Front Pard overlaps with Front
Oorch. 8owever& this does not give us information about the other sections of
the diagram. $tatement !" 01/-) is not sufficient.
%he correct answer is 0.
#%.
%his is a problem that involves three overlapping sets. 0 helpful wa, to visualize this
is to draw a Renn diagram as follows;
)ach section of the diagram represents a different group of people. $ection a
represents those residents who are members of onl, club a. $ection b represents
those residents who are members of onl, club b. $ection c represents those residents
who are members of onl, club c. $ection w represents those residents who are
members of onl, clubs a and b. $ection " represents those residents who are
members of onl, clubs a and c. $ection y represents those residents who are
members of onl, clubs b and c. $ection & represents those residents who are
members of all three clubs.
%he information given tells us that a * b * c . 40. /ne wa, of rephrasing
the 4uestion is as follows; 3s " V 0 Q Gecall that " represents those residents who are
member of fitness clubs 0 and 7 but not +".
$tatement 1" tells us that & . !. 0lone& this does not tell us an,thing about "& which
could& for example& be 0 or 10& among man, other possibilities. %his is clearl, not
sufficient to answer the 4uestion.
$tatement !" tells us that w * y # '. %his alone does not give us an, information
about ", which& again could be 0 or a number of other values.
3n combining both statements& it is temptin+ to assert the following.
5e (now from the 4uestion stem that a E b * c . 40. 5e also (now
from statement one that & # !. Finall,& we (now from statement two that w * y # '.
5e can use these three pieces of information to write an e4uation for all 55 residents
as follows;
a E b * c * w * " * y * & . 55.
3a E b * c4 * " * 3w * y4 * 3&4 . 55.
40 E " E ' E ! # 55
" # 5
%his would suggest that there are 5 residents who are members of both fitness clubs
0 and 7 but not +.
5owever& this assumes that all 55 residents belon+ to at least one fitness club. Pet&
this fact is not stated in the problem. 3t is possible then& that 5 of the residents are not
members of any fitness club. %his would mean that 0 residents are members of
fitness clubs 0 and 7 but not +.
5ithout (nowing how man, residents are not members of any fitness club& we do
not have sufficient information to answer this 4uestion.
%he correct answer is ); $tatements 1" and !" %/<)%8)G are -/% sufficient.
20.
From 1& 16 students stud, both French and Japanese& so 16@0.04#400 students stud,
French& combine Dat least 100 students stud, JapaneseD& insufficient.
From !& we can (now that& 10. Japanese stud,ing students#4. French stud,ing
students.
0pparentl,& more students at the school stud, French than stud, Japanese.
0nswer is +
(!.
$tatement 1 is sufficient.
For !& 3#0E+E7*0+*07*+7E0+7& we (now 0& + &7& 0+& 07& +7& but we donUt
(now 3& so& 0+7 cannot be resolve out.
0nswer is 0
(#.
1". %he total number is 1!0& then the number is; 1!0W!@2W1*2@5"#2!
!". 40 students li(e beans& then total number is 40@1@2#1!0& we can get the same
result.
0nswer is C
Per&enta8es
1. Oercentage problems involving unspecified amounts can usuall, be solved more
easil, b, using the number 100. 3f 0rthur>s fortune was originall, X100& each of
his children received X!0.
1et>s see what happened to each X!0 investment in the first ,ear;
0lice; X!0 E X10 profit # X20
+ob; X!0 E X10 profit # X20
7arol; X!0 E X10 profit # X20
Cave; X!0 * X' loss # X1!
)rrol; X!0 * X' loss # X1!
5e continue on with our new amounts in the second ,ear;
0lice; X20 E X2 profit # X22
+ob; X20 E X2 profit # X22
7arol; X20 * X1' loss # X1!
Cave; X1! E X2 profit # X15
)rrol; X1! * X1! # 0
0t the end of two ,ears& X22 E X22 E X1! E X15 # X62 of the original X100
remains.
%he correct answer is 0.
!. %his is a weighted average problemL we cannot simpl, average the percentage of
silver cars for the two batches because each batch has a different number of cars.
%he car dealership currentl, has 40 cars& 20. of which are silver. 3t receives '0 new
cars& 60. of which are silver the 40. figure given in the problem refers to cars
which are not silver". -ote that the first batch represents 1@2 of the total cars and the
second batch represents !@2 of the total cars. Out differentl,& in the new total group
there is 1 first,batch car for ever, ! second,batch cars.
5e can calculate the weighted average& weighting each percent according to the ratio
of the number of cars represented b, that percent;
0lternativel,& ,ou can calculate the actual number of silver cars and divide b, the
total number of cars. 400.2" E '00.6" # 1! E 4' # 60. 60@1!0 # 50..
%he correct answer is ).
2. -otice that OaulUs income is expressed as a percentage of GexUs and that the other
two incomes are expressed as a percent of OaulUs. 1etUs assign a value of X100 to
5eighted average #
!20." E
#60."
(
#
150.
2
# 50.
GexUs income. OaulUs income is 40. less than Gex>s income& so 0.6"X100" #
X60. NuentinUs income is !0. less than Oaul>s income& so 0.'"X60" # X4'. $amUs
income is 40. less than Oaul>s income& so 0.6"X60" # X26. 3f Gex gives 60. of
his income& or X60& to $am& and 40. of his income& or X40& to Nuentin& then; $am
would have X26 E X60 # X66 and Nuentin would have X4' E X40 # X''. NuentinUs
income would now be X''@X66 # 11@1! that of $am>s.
%he correct answer is 0.
4.
1etUs denote the formula for the mone, spent on computers as p6 # b& where
p # price of computers
6 # 4uantit, of computers
b # budget
5e can solve a percent 4uestion that doesnUt involve actual values b, using smart
numbers. 1etUs assign a smart number of 1000 to last ,earUs computer budget b" and
a smart number 100 to last ,earUs computer price p". 1000 and 100 are eas, numbers
to ta(e a percent of.
%his ,earUs budget will e4ual 1000 A 1.6 # 1600
%his ,earUs computer price will e4ual 100 A 1.! # 1!0
-ow we can calculate the number of computers purchased each ,ear& 6 # b@p
-umber of computers purchased last ,ear # 1000@100 # 10
-umber of computers purchased this ,ear # 1600@1!0 # 12 1@2 while 1@2 of a
computer doesnUt ma(e sense it wonUt affect the calculation"
p 6 b
%his
Pear
100 10 1000
1ast
Pear
1!0
12
1@2
1600
%he 4uestion is as(ing for the percent increase in 4uantit, from last ,ear to this ,ear #
%his
4uestion could also have been solved algebraicall, b, converting the percent
increases into fractions.
1ast ,ear; p6 # b& so 6 # b@p
%his ,ear; 6@5"p""" # '@5"b
3f we solve for " this ,ear>s 4uantit,"& we get " # '@5"5@6"b@p or 4@2"b@p
new F old
old
A 100. #
12 1@2 F
10
10
A 100. # 22 1@2.
3f this ,ear>s 4uantit, is 4@2 of last ,ear>s 4uantit, b@p"& this represents a 22 1@2.
increase.
%he correct answer is 0.
5. %his problem can be solved most easil, with the help of smart numbers. 5ith
problems involving percentages& 100 is t,picall, the YsmartestU of the smart
numbers.
3f we assume that toda,Us population is 100& next ,ear it would be 1.1 A 100 #
110& and the following ,ear it would be 1.1 A 110 # 1!1. 3f this is double the
population of one ,ear ago& the population at that time must have been 0.5 A 1!1
# 60.5. +ecause the problem see(s the 9closest: answer choice& we can round
60.5 to 60.
3n this scenario& the population has increased from 60 to 100 over the last ,ear& a
net increase of 40 residents. %o determine the percentage increase over the last
,ear& divide the net increase b, the initial population; 40@60 # 4@6 # !@2& or
roughl, 67..
For those who prefer the algebraic approach; let the current population e4ual p.
-ext ,ear the population will e4ual 1.1p& and the following ,ear it will e4ual 1.1 A
1.1p # 1.!1p. +ecause the 4uestion as(s for the closest answer choice& we can
simplif, our algebra b, rounding 1.!1p to 1.!p. 8alf of 1.!p e4uals 0.6p. %he
population increase would be e4ual to 0.4p@0.6p # 0.4@0.6 # !@2& or roughl, 67..
%he correct answer is C.
6.
%o solve this problem& first find the wholesale price of the shirt& then compute the
price re4uired for a 100. mar(up& then subtract the X45 initial retail price to get the
re4uired increase.
1et " e4ual the wholesale price of the shirt. %he retailer mar(ed up the wholesale
price b, '0. so the initial retail price is " E '0. of "". %he following e4uation
expresses the relationship mathematicall,;
" E 0.'0" # 45
1.'" # 45
" # 45@1.'
" # 450@1'
" # !5
$ince the wholesale price is X!5& the price for a 100. mar(up is X50. %herefore the
retailer needs to increase the X45 initial retail price b, X5 to achieve a 100. mar(up.
%he correct answer is ).
7.
5e can solve this as a R37 Rariable 3n answer 7hoices" and plug in values for " and
r.
7
cents per person per
mile
10
8 Z of miles !0
$ince there are 2 people& the taxi driver will charge them 20 cents per mile.
$ince the, want to travel !0 miles& the total charge no discount" would be 20"!0" #
600.
5ith a 50. discount& the total charge will be 200 cents or 2 dollars.
3f we plug r # 10 and " # !0 into the answer choices& the onl, answer that ,ields 2
dollars is C.
%he correct answer is C.
'.
+ob put !0 gallons of gasohol into his car>s gas tan(& consisting of 5. ethanol and
65. gasoline. 7hemical ?ixture 4uestions can be solved b, using a mixture chart.
$B+$%0-7)$ 0?/B-% O)G7)-%0<)
)%80-/1 1 5.
<0$/13-) 16 65.
%/%01$ !0 100.
%his chart indicates that there is 1 gallon of ethanol out of the full !0 gallons& since
5. of !0 gallons is 1 gallon.
-ow we want to add " gallons of ethanol to raise it to a 10. ethanol mixture. 5e can
use another mixture chart to represent the altered solution.
$B+$%0-7)$ 0?/B-% O)G7)-%0<)
)%80-/1 1 E " 10.
<0$/13-) 16 60.
%/%01$ !0 E " 100.
%herefore& the following e4uation can be used to solve for ";
1 E " # ! E 0.1"
0.6" # 1
1 E "
!0 E "
# 10.
" # 10@6
%he correct answer is 7.
6. -oting that 65. is ver, close to !@2& we ma, approximate the original expression
as follows;
1@2 E 0.4 E 65. /riginal expression
1@2 E 0.4 E !@2 7lose approximation
1 E 0.4
1.4
%he correct answer is C.
10.
First& letUs find the initial amount of water in the tan(;
%otal mixture in the tan( #1@4 A capacit, of the tan(" # 1@4" A !4 # 6 gallons
7oncentration of water in the mixture # 100. F concentration of sodium chloride" #
100. F 40. # 60.
3nitial amount of water in the tan( # 60. A total mixture"# 0.6 A 6 # 2.6 gallons
-ext& letUs find the amount and concentration of water after ! hours;
0mount of water that will evaporate in ! hours # rate of evaporation"time" # 0.5!"
# 1 gallon
Gemaining amount of water # initial amount F evaporated water # 2.6 F 1 # !.6
gallons
Gemaining amount of mixture # initial amount F evaporated water # 6 F 1 # 5 gallons
%he correct answer is 7.
11.
/ne of the most effective wa,s to solve problems involving formulas is to pic(
numbers. -ote that since we are not given actual values but are as(ed to compute
onl, the relative change in the useful life& we can select eas, numbers and plug them
7oncentration of water in the mixture in ! hours #
remaining water
remaining mixture
A 100.
which e4uals;
!.6
5
A 100. #
5!.
into the formula to compute the percentage increase. 1etUs pic( d # 2 and h # ! to
simplif, our computations;
+efore the change; d # 2& h # !L u # '"2"@!
!
# !4@4 # 6
0fter the change; d # !"2"# 6& h #!@! #1L u # '"6"@1
!
# 4'
Finall,& percent increase is found b, first calculating the change in value divided b,
the original value and then multipl,ing b, 100;
4' F 6"@6 # 4!@6" # 7
7"100" # 700.
%he correct answer is C.
1!. $ince there are variables in the answer choices& as well as in the 4uestion stem&
one wa, to approach this problem is to pic( numbers and test each answer choice.
5e (now that " is m percent of !y& so pic( values for m and y& then solve for ".
y # 100
m # 40
" is m percent of !y& or " is 40 percent of !00& so " # 0.40"!00" # '0.
$o& for the numbers we are using& m is what percent of "Q 5ell& m # 40& which is
half of " # '0. %hus& m is 50 percent of ". %he answer choice that e4uals 50 will
be the correct choice.
0" y@!00 # 100@!00 # 0.5 5G/-<
+" !y # !"100" # !00 5G/-<
7" 50y # 50"100" # 5000 5G/-<
C" 50@y # 50@100 # 0.5 5G/-<
)" 5000@y # 5000@100 # 50 7/GG)7%
0lternativel,& we can pursue an algebraic solution.
5e are given the fact that " is m percent of !y& or " # m@100"!y" # my@50. $ince
the 4uestion as(s about m 9m is what percent of "Q:"& we should solve this
e4uation for m to get m # 50@y""".
Outting the 4uestion 9m is what percent of "Q: into e4uation form& with the word
9/nswer: as a placeholder& m # /nswer@100""".
-ow we have two e4uations for m. 3f we set them e4ual& we can solve for the
9/nswer.:
/nswer@100""" # 50@y"""
/nswer@100" # 50@y"
/nswer # 5000@y
%he correct answer is ).
12. %he easiest wa, to solve this problem is to use the R37 method of substituting
actual numbers for x and ,. %he problem as(s us to ta(e ". of y and increase it b,
".. $ince we are dealing with percentages& and the whole y" is neither given to
us nor as(ed of us& let>s set y # 100 and " # 10. -ote that this is a variation on the
t,pical method of pic(ing small numbers in R37 problems.
10. of 100 is 10. 3ncreasing that 10 b, 10. gives us 10 E 1 # 11. %herefore 11 is
our target number. 1et>s test each answer choice in turn to see which of them
matches our target number.
0" 100"y E " # 10010"100" E 10 which doesn>t e4ual 11.
+" "y E "@100 # 10100" E 10@100 which doesn>t e4ual 11.
7" 100"y E "@100 # 10010"100" E 10@100 which doesn>t e4ual 11.
which doesn>t e4ual 11.
# 11
%he correct answer is ).
"y
C" 100"y E
100
10100"
# 10010"100" E
100
"y " E 100"
)"
#
10000
10" 100" 10 E
100"
#
10000
10 E 100
10
14.
First& determine the total cost of the item at each store.
$tore 0;
X60 ?$GO"
E X1! E !0. mar(*up # 0.!0 A X60"
X7!.00 purchase price"
E X2.60 E 5. sales tax # 0.05 A X7!"
X75.60 total cost"
$tore +;
X60 ?$GO"
E X1' E 20. mar(*up # 0.20 A X60"
X7'.00 regular price"
F X7.'0 F10. sale # F0.10 A X7'"
X70.!0 current purchase price"
E X2.51 5. sales tax # 0.05 A X70.!0"
X72.71 total cost"
%he difference in total cost& subtracting the $tore + cost from the $tore 0 cost& is thus
X75.60 * X72.71 # X1.'6.
%he correct answer is C.
15.
<iven an initial deposit of X1&000& we must figure out the ending balance to calculate
the total percent change.
0fter the first ,ear& $am>s account has increased b, X100 to X1&100.
0fter the second ,ear& $am>s account again increased b, 10.& but we must ta(e 10.
of X1&100& or X110. %hus the ending balance is X1&!10 X1&100 E X110".
%o calculate the percent change& we first calculate the difference between the ending
balance and the initial balance; X1&!10 F X1&000 # X!10. 5e divide this difference b,
the initial balance of X1&000 and we get X!10@X1&000 # .!1 # !1..
%he correct answer is 7.
16.
Oroblems that involve successive price changes are best approached b, selecting a
smart number. 5hen the problem deals with percentages& the most convenient smart
number to select is 100. 1etUs assign the value of 100 to the initial price of the
painting and perform the computations;
/riginal price of the painting # 100.
Orice increase during the first ,ear # !0. of 100 # !0.
Orice after the first ,ear # 100 E !0 # 1!0.
Orice decrease during the second ,ear # 15. of 1!0 # 1'.
Orice after the second ,ear # 1!0 F 1' # 10!.
Final price as a percent of the initial price # 10!@100" # 1.0! # 10!..
%he correct answer is 0.
17. 5e can solve this 4uestion as a R37 Rariable in answer choices" b, plugging in
values for "& y and &;
"
percent
mar(*up
1st"
10
y
percent
discount
!nd"
!0
&
original
price
100
3f a X100 item is mar(ed up 10. the price becomes X110. 3f that same item is then
reduced b, !0. the new price is X''.
3f we plug " # 10& y # !0& & # 100 into the answer choices& onl, answer choice 0"
gives us '';
0lternativel, we could have solved this algebraicall,.

5e can combine these as; &1 E "@100"1 F y@100".
10&000100" E 100100"10 F!0" F 10"!0"
100"
10&000
# ''
0 price mar(up of " percent is the same as multipl,ing the price & b, 1 E
"
100
"
.
0 price discount of y percent is the same as b, multipl,ing b, 1 F
y
100
"
.
%his can be simplified
to;
10&000& E 100&" F y" F
"y&
10&000
%he correct answer is 0.
1'. 3f p is the price that the shop originall, paid for the cloc(& then the price that
the collector paid was 1.!p to ,ield a profit of !0.". 5hen the shop bought bac(
the cloc(& it paid 50. of the sale price& or .5"1.!"p # .6p. 5hen the shop sold the
cloc( again& it made a profit of '0. on .6p or 1.'".6"p . 1.0'p.
%he difference between the original cost to the shop p" and the bu,*bac( price
.6p" is X100.
%herefore, p F .6p # X100. $o& .4p # X100 and p # X!50.
3f the second sale price is 1.0'p& then 1.0'X!50" # X!70. -ote; at this point& if
,ou recognize that 1.0'p is greater than X!50 and onl, one answer choice is
greater than X!50& ,ou ma, choose not to complete the final calculation if ,ou are
pressed for time."
%he correct answer is 0.
16. 5e are told that the bo,s of Jones )lementar, ma(e up 40. of the total of x
students.
%herefore; 9 of boys # .4"
5e are also told that ". of the 9 of boys is 60.
%hus& using "@100 as ".;
"@100" A 9 of boys" # 60
$ubstituting for 9 of boys from the first e4uation& we get;
"@100" A .4" # 60
.4"
!
" @ 100 # 60
.4"
!
# 6&000
"
!
# !!&500
" # 150
0lternativel,& we could have plugged in each answer choice until we found the
correct value of ". +ecause the answer choices are ordered in ascending order& we
can start with answer choice 7. %hat wa,& if we get a number too high& we can
move to answer choice + and if we get a number too low& we can move to answer
choice C.
<iven an " of !!5 in answer choice 7& we first need to ta(e 40.. 5e do this b,
multipl,ing b, .4.
.4 A !!5 # 60
-ow& we need to ta(e ". of this result. 0gain& ". is Hust "@100& in this case
!!5@100 or !.!5.
%hus ". of our result is; !.!5 A 60 # !0!.5
%his is too high so we tr, answer choice +. Following the same series of
calculations we get;
.4 A 150 # 60
". # 150@100 # 1.5
1.5 A 60 # 60
%his is the result we are loo(ing for& so we are done.
%he correct answer is +.
!0. %he dress has three different prices throughout the course of the problem; the
original price which we will call ""& the initial sales price X6'" and the final
selling price which we will call y". 3n order to answer the 4uestion& we must find
the other two prices " and y.
0ccording to the problem& the original price" A '5. # initial sales price # X6'&
therefore " # 6' @ 0.'5. 8ow can we do this arithmetic efficientl,Q 0.'5 is the same
as '5@100 and this simplifies to 17@!0. 6' @ 17@!0" # 6' A !0@17". 17 goes into 6'
four times& so the e4uation further simplifies to 4 A !0 # '0. %he original price was
therefore X'0.
0ccording to the problem& the initial sales price A 1!5. # final selling price&
therefore 6' A 1!5. # y. ?ultipl,ing b, 1!5. is the same thing as finding !5. of
6' and adding this figure to 6'. !5. of 6' is 17& so the final selling price was X6' E
X17 # X'5.
%he difference between the original and final prices is X'5 F X'0 # X5.
%he correct answer is C.
!1.
3f we denote the amount of mone, owned b, Jennifer as ( and that owned b, +rian as
b& we can create two e4uations based on the information in the problem. First&
Jennifer has 60 dollars more than +rian; ( # b E 60.
$econd& if she were to give +rian 1@5 of her mone,& she would have ( F 1@5"( # 4@5"(
dollars. +rian would then have b E 1@5"( dollars. %herefore& since +rianUs amount of
mone, would be 75. of JenniferUs& we can create another e4uation; b E 1@5"( #
0.75"4@5"(& which can be simplified as follows;
b E 1@5"( # 0.75"4@5"(
b E 1@5"( # 2@4"4@5("
b E 1@5"( # 2@5"(
b # 2@5"( F 1@5"(
b # !@5"(
$ubstitute this expression for b bac( into the first e4uation& then solve for (;
( # b E 60
( # !@5"( E 60
( F !@5"( # 60
2@5"( # 60
( # 60"5@2" # 100
%herefore& Jennifer has 100 dollars.
%he correct answer is +.
!!.
3n this case& the average computer price three ,ears ago represents the original
amount. %he original amount # '0. of X700 or X560. %he chan+e is the difference
between the original and new prices # X700 F X560 # X140. . change # !5..%he
correct answer is 7.
!2. %o determine the total capacit, of the pool& we need to first determine the
amount of water in the pool before the additional water is added. 1et>s call this
amount b.
0dding 200 gallons represents a 20. increase over the original amount of water in
the pool. %hus& 200 # 0.20b. $olving this e4uation& ,ields b # 1000. %here are 1000
gallons of water originall, in the pool.
0fter the 200 gallons are added& there are 1200 gallons of water in the pool. %his
represents '0. of the pool>s total capacit,& -.
1200 # .'0-
1200 # 4@5"-
12005@4" # -
- # 16!5
%he correct answer is )
!4. 3nstead of performing the computation& set up a fraction and see if terms in
the numerator cancel out terms in the denominator; -otice that the 16 cancels out the
two 4s and that the 1000 in the numerator cancels the 1000 in the denominator. %hus&
we are left with ! A 2 A 2 # 1'. %his is the e4uivalent of 1.' A 10. %he correct answer
is C.
!5. 0.25 is greater than 0.007 so it must represent more than 100. of .007. %his
eliminates answer choices 0& +& and 7. Bse benchmar(s values to help ,ou arrive at
the final answer;
100. of 0.007 # 0.007 %a(ing 100. of a number is the e4uivalent of multipl,ing b,
1."
500. of 0.007 # 0.025 %a(ing 500. of a number is the e4uivalent of multipl,ing b,
5."
5000. of 0.007 # 0.25 %a(ing 5000. of a number is the e4uivalent of multipl,ing
b, 50."
%he correct answer is ).
!6.
5e are as(ed to find the dollar change in the price of the propert, during the second
,ear. $ince we (now the percent changes in each ,ear& we will be able to answer the
4uestion if we (now the price of the propert, in an, of these ,ears& or& alternativel,& if
we (now the dollar change in the propert, price in an, particular ,ear.
1" $BFF373)-%; $ince we (now the price of the propert, at the end of the three*
,ear period& we can find the original price and determine the price increase during the
second ,ear. 1et p denote the original price of the propert,;
p1.1"0.'"1.!5" # !!&000
1.1p # !!&000
p # !0&000
Orice of the propert, after the first ,ear; !0&0001.1" # !!&000
Orice of the propert, after the second ,ear; !!&0000.'" # 17&600
Cecrease in the propert, price # !!&000 F 17&600 # 4&400
!" $BFF373)-%; %his information is sufficient to create an e4uation to find the
original price and determine the dollar change during the second ,ear;
Orice at the end of the first two ,ears; p1.1"0.'" # 0.''p
Orice decrease over the two*,ear period # original price F ending price # p F 0.''p #
0.1!p
Orice increase in the third ,ear # price at the end of two ,ears"!5." # 0.''p0.!5"
#0.!!p
%hus& since we (now that the difference between the price decrease over the first two
,ears and the price increase over the third ,ear was X!&000& we can create the
following e4uation and solve for p;
0.!!p F 0.1!p # !&000
0.1p # !&000
p # !0&000
Orice of the propert, after the first ,ear; !0&0001.1" # !!&000
Orice of the propert, after the second ,ear; !!&0000.'" # 17&600
Cecrease in the propert, price # 4&400
%he correct answer is C.
!7.
1et i be the salesman>s income& let s be the salar,& and let c be the commission. From
the 4uestion stem we can construct the following e4uation;
i # s E c
5e are as(ed whether s accounts for more than half of i. 5e can thus rephrase the
4uestion as D3s s greater than cQD
$BFF373)-%; %his allows us to construct the following e4uation;
1.1i # s E 1.2c
$ince we alread, have the e4uation i # s E c& we can subtract this e4uation from the
one above;
.1i # .2c
-otice that the s>s cancel each other out when we subtract. 5e can isolate the c b,
multipl,ing both sides b, 10@2 the reciprocal of .2 or 2@10";
1@10"i # 2@10"c
1@10"i A 10@2" # 2@10"c A 10@2"
1@2"i # c
%herefore c is one*third of the salesman>s income. %his implies that the salar, must
account for two*thirds of the income. %hus& we can answer definitivel, that the salar,
accounts for more than half of the income.
3-$BFF373)-%; )ither s * c # .5s or c * s # .5s. 7oupled with our (nowledge
that s and c must add to 100. of the salesman>s income& we can sa, that one
of the two is worth 75. of the income and the other is worth !5.. 8owever&
we don>t (now which is the bigger number; s or c.
%he correct answer is 0.
!'. 1et>s assume m is the number of hot dogs sold in ?a,& and H is the number sold in
June. 5e (now that the vendor sold 10. more hot dogs in June than in ?a,& so we
can set up the following relationship; H # 1.1m. 3f we can find the value for one of
these variables& we will be able to calculate the other and will& therefore& be able to
determine the value of m E (.
1" $BFF373)-%; 3f the vendor sold !7 more hot dogs in June than in ?a,& we can
sa, H # !7 E m. -ow we can use the two e4uations to solve for H and m;
H # 1.1m
H # !7 E m
$ubstituting 1.1m in for H gives;
1.1m # !7 E m
.1m # !7
m # !70
H # m E !7 # !67
$o the total number of hot dogs sold is m E H # !70 E !67 # 567.
!" 3-$BFF373)-%; 5hile (nowing the percent increase from ?a, to Jul, gives us
enough information to see that the number of hot dogs sold each month increased& it
does not allow us to calculate the actual number of hot dogs sold in ?a, and June.
For example& if the number of hot dogs sold in ?a, were 100& then the number sold
in June would be 1.1100" # 110& and the number sold in Jul, would be 1.!100" #
1!0. %he total number sold in ?a, and June would be 100 E 110 # !10. 8owever&
the number sold in ?a, could Hust as easil, be !00& in which case the number sold in
June would be 1.1!00" # !!0& and the number sold in Jul, 1.!!00" # !40. %he total
number for ?a, and June in this case would be !00 E !!0 # 4!0.
%he correct answer is 0.
!6. 5e can determine the sales revenue that the sales associate generated b,
anal,zing her commission earnings for the wee(.
$BFF373)-%; %he sales associate earned a total of X1500 in commission last
wee(. 5e (now that on the first X10&000 in sales revenue& the associate earns
'. or X'00 in commission. %his means that the associate earned X700 in
additional commission. $ince this additional commission is calculated based
on a 10. rate& the sales associate must have generated an additional X7000
worth of sales revenue. %hus& we (now from statement 1 that the sales
associate generated X10&000 E X7000 # X17&000 in sales revenue last wee(.
$tatement 1 alone is sufficient.
$BFF373)-%; %he sales associate was eligible for the 10. commission rate
on X7000 worth of sales. $ince the 10. rate onl, (ic(s in after the first
X10&000 in sales& this means that the sales associate generated X7000 in sales
revenue above the X10&000 threshold. %hus& we (now from statement ! that
the sales associate generated X10&000 E X7000 # X17&000 in sales revenue last
wee(. $tatement ! alone is sufficient.
%he correct answer is C.
20.
5e are told that the team won y games out of a total of " games. %hen we are as(ed
for the value of y. 5e cannot rephrase the 4uestion in an, useful wa,& so we must
proceed to the statements.
1" 3-$BFF373)-%; 5e are told that if the team had lost two more games& it would
have won !0. of its games for the season. %his implies that it would have lost '0.
of its games under this condition. %he number of games that the team lost is " F y. $o
we can construct the following e4uation;
100" F 100y E !00 # '0"
!0" E !00 # 100y
" F y E
!
"
#
'0
100
" E 10 # 5y
%his is not sufficient to tell us the value of y.
!" 3-$BFF373)-%; 5e are told that if the team had won three more games& it would
have lost 20. of its games for the season. %his implies that it would have won 70.
of its games under this condition. $o we can construct the following e4uation;
100y E 200 # 70"
10y E 20 # 7"
%his is not sufficient to tell us the value of y.
1" 0-C !" $BFF373)-%; 5e now have two different e4uations containing onl, the
same two un(nowns. 5e can use these e4uations to solve for y though recall that ,ou
should onl, ta(e the calculation far enough that ,ou (now ,ou can finish& since this is
data sufficienc,";
7" F 20 # 10y
" E 10 # 5y
7" F 20 # 10y
!" E 10 # 5y"
7" F 20 # 10y
!" E !0 # 10y
$ubtract bottom e4uation from top;
5" F 50 # 0
5" # 50
" # 10
3f " # 10& then all we need to do is plug 10 in for " in one of our e4uations to find the
value of y;
" E 10 # 5y
10 E 10 # 5y
!0 # 5y
4 # y
%he correct answer is 7.
21.
1et " represent the compan,Us profits in 166!& y represent the profits in 1662& and &
represent the profits in 1664. $ince the profits in 1662 were !0. greater than in 166!&
y # 1.!"& or y@" # 1.!. $imilarl,& since the profits in 1664 were 10. greater than in
1662& & # 1.1y& or &@y # 1.1. $ince we have ratios relating the three variables& (nowing
the profits from an, of the three ,ears will allow us to calculate the profits in 166!.
$o& the rephrased 4uestion is; 95hat is "& y& or &Q:
y E 2
"
#
70
100
1" $BFF373)-%; %his statement tells us that & # 100&000 E y. 5e also (now the
ratio of & to y; &@y # 1.1. 7ombining the two e4uations and substituting for & gives;
&@y # 1.1
100&000 E y"@y # 1.1
100&000 E y # 1.1y
100&000 # .1y
y # 1&000&000
%he profits in 1662 were X1&000&000. $ince we (now y # 1.!"& this information is
sufficient to determine the profits in 166!.
!" 3-$BFF373)-%; %his tells us that the ratio of & to " is; &@" # 2.66@2 # 1.2!.
8owever& we alread, (now from information given in the 4uestion that;
y # 1.!" and & # 1.1y
& # 1.11.!""
& # 1.2!"
&@" # 1.2!
$o& statement !" gives no new information.
%he correct answer is 0.
2!.
3n order to determine the percent discount received b, Jamie& we need to (now two
things; the regular price of the des( and the sale price of the des(. 0lternativel,& we
could calculate the percent discount from the price reduction and either the regular
price or the sale price.
1" 3-$BFF373)-%; %his statement tells us the regular price of the des( at 0lUs& but
provides no information about how much Jamie actuall, paid for the des( during the
annual sale.
!" 3-$BFF373)-%; %his statement tells us how much the price of the des( was
reduced during the sale& but provides no information about the regular price. For
example& if the regular price was X6010& then the discount was onl, 10.. /n the
other hand& if the regular price was X60!& then the discount was nearl, 100..
1" 0-C !" 3-$BFF373)-%; 0t first glance& it seems that the statements together
provide enough information. $tatement 1" seems to provide the regular price of the
des(& while statement !" provides the discount in dollars.
8owever& pa, attention to the words 9rounded to the nearest percent: in statement
1". %his indicates that the regular price of the des( at 0lUs is 60. of the ?$GO& plus
or minus 0.5. of the ?$GO. Gather than clearl, stating that the regular price is 0.60"
X!000" # X1!00& this statement gives a range of values for the regular price; X1!00
plus or minus X10 0.5. of !000"& or between X1160 and X1!10.
3f the regular price was X1160& then the discount was X601@X1160" A 100. # 50.5.
,ou can actuall, see that this is greater than 50. without calculating".
3f the regular price was X1!10& then the discount was X601@X1!10" A 100. # 46.7.
,ou can actuall, see that this is less than 50. without calculating".
%he uncertaint, about the regular price means that we cannot answer with certaint,
whether the discount was more than 50. of the regular price.
%he correct answer is ).
22. 0ccording to the 4uestion stem&
total cost # fixed cost E variable cost
:
t
# :
f
E :
v

%he 4uestion is as(ing for the percent change of the total cost of production of item K
in Januar,. 7learl, if we (new the total cost of producing K before Januar, and then
in Januar,& we could calculate the percent change. From the 4uestion& however& it
doesnUt seem li(e we will be provided with this information.
1" 3-$BFF373)-%; $ince the total cost of production is also the sum of the fixed
and variable costs& it would stand to reason that we should be able to calculate the
percent change to the total cost if we (new the percent change of the fixed and
variable costs.
8owever& it is not that simple. 5e cannot simpl, average the percent change of the
fixed and variable costs to find the percent change of the total cost of production.
%wo percents cannot be averaged unless we (now what relative portions the,
represent.
1etUs use numbers to illustrate this point. 3n the first set of numbers in the table
below& the fixed cost is 100 times the size of the variable cost. -otice that the percent
change of the total cost of production is almost identical to the percent change of the
fixed cost. 3n the second set of numbers& the fixed and variable costs are identical.
-otice that the percent change of the total cost of production is exactl, the average of
the percent change of the variable cost and the fixed cost 4. is the average of 12.
and *5.".
+efore 3n Januar, %/%01$
:
f
100 112 E 12.
:
v
1 .65 * 5.
:
t
101 112.65 [ E12.
:
f
100 112 E 12.
:
v
100 65 * 5.
:
t
!00 !0' E 4.
!" 3-$BFF373)-%; 1ac(ing information about the percent change of the fixed cost&
we cannot solve.
1" 0-C !" $BFF373)-%; Bsing the two statements& we not onl, have the percent
changes of the fixed and variable percents& but we also (now the relative portions
the, represent.
3f the fixed cost before Januar, was five times that of the variable cost& we can
calculate the percent change to the cost of production using a weighted average;
0lternativel, if we
tr, different values for :
f
and :
v
that each maintain the 5;1 ratio& we will come up
with the same result. %he cost of production increased in Januar, b, 10..
%he correct answer is 7.
(4.
%his problem can be conceptualized with the help of smart numbers and some simple
algebra. +ecause we are wor(ing with percentages and are given no real values& it is
sensible to begin with the assumption that there are 100 attendees at the part,.
%herefore& there must be 40 females and 60 males.
1et m e4ual the number of men who arrived solo& w e4ual the number of women who
arrived solo& and p e4ual the number of attendees who arrived with a companion p@!
would e4ual the number of pairs". Bsing our smart numbers assumption& m E w E p #
100. %his 4uestion might therefore be rephrased& 95hat is m E wQ: or 95hat is 100 F
pQ:
1" $BFF373)-%; <iven 60 male guests& $tatement 1" tells us that 20 arrived with a
companion. %herefore& 20 men and 20 women arrived in pairs. Gecall that p e4uals
Oercent change of :
t
#
5 A percent change of :f" E 1 A percent change of
:v"
6
Oercent change of :
t
#
5 A 12." E 1 A *
5."
6
# 10.
the total number of guests arriving in pairs& so p # 60. <iven that 100 F p is sufficient
to solve our problem& $tatement 1" is sufficient; 40 individuals 40. of the total
number of guests" arrived at the part, alone.
!" $BFF373)-%; %his statement tells us that
.!5m E w" # w
.!5m E .!5w # w
.!5m # .75w
m # 2w
Further& observe that the total number of women at the part, would e4ual the number
arriving solo plus the number arriving with a companion;
40 # w E p@!
'0 # !w E p
Finall,& recall that m E w E p # 100.
5e now have three e4uations with three un(nowns and are able solve for m& w and p&
so $tatement !" is sufficient. 5hile it is unnecessar, to complete the algebra for this
data sufficienc, problem& witness;
$ubstituting 2w for m in the e4uation m E w E p # 100 ,ields 4w E p # 100.
!w E p # '0
4w E p # 100
$ubtracting the first e4uation from the second ,ields
!w # !0
w # 10
p # 60
m # 20
%he correct answer is C.
25. 3n order to answer this 4uestion& we must determine the value of a
b
.
3-$BFF373)-%; %his tells us that b # !a. 8owever& this does not allow us to
solve for a
b
.
3-$BFF373)-%; %his tells us that .5b # a. %his can be rewritten as b # !a.
8owever& this does not allow us to solve for a
b
.
0-C !" 3-$BFF373)-%; $ince both statements provide the exact same
information& ta(ing the two together still does not allow us to solve for a
b
.
%he correct answer is ).
26.
Fat in mil( is xW1.& ,W!. and zW2.& respectivel,.
$o we have the e4uation; xW1.E,W!.EzW2.#xE,Ez"W1.5.
$implif, the e4uation& we can obtain that x#,E2z
(7.
For 1& the tip for a X15 bill will be X!& which is less than X15W15.#!.!5L the tip for a
X!0 will be X4& which is greater than X15W15.#!.!5. 3nsufficient.
For !& tips is X'& means the tens digit of the bill is 4& and the largest possible value of
the bill is X46. X'V46W15.#7.25. $ufficient alone.
0nswer is +
(".
1et their hourl, wage are x and ,.
%herefore& after the increases& the difference between their wages is 1.06x*1.06,
From 1& x*,#5& we can solve out 1.06x*1.06,
From !& x@,#4@2& insufficient.
0nswer is 0
(%.
1et ? be the number of the cameras produced in 1665.
S?@1E,."T@1Ex."#1000
?#1000E10xE10,Ex,@10
Mnowing that xE,E x,@100 # 6.!& ? can be solve out.
0nswer is +
40.
1et x and , be the numbers of the male and female students.
7ombined 1 and !& 25.KE!0.P#!5.KEP"
10.K#5.P
P#!K
K@KEP"#K@2K#1@2
0nswer is 7
4!.
For statement 1& xV75& then ,V1.1W75V75
For statement !& x#10& ,#!0 can fulfill the re4uirement& but the ,\75
0nswer is 0
4#.
/bviousl,& !0.menE10.womenV10.menEwomen".
0nswer is +
4(.
5e (now that; revenue#gross profit E expense
1". Gevenue#1@2 expenseEexpense#4@2 expense& gross profit is 1@4 of its revenue.
!". <ross profit#revenue ]7expense#1@4 revenue& gross profit is 1@4 of its revenue.
0nswer is C
44.
1". %he discount of the most expensive item was !0.W50#X10& the discount of the
next expensive item was 10.W!0#!
let the regular price of the other item is x& then x\#!0 and discount is 10.x
$o& total discount is 10E!E0.1K& sum of the regular price is 50E!0EK
%hen& total discount@sum of the regular prices was;
1!E0.1x"@70Ex"#7E0.1KE5"@70Ex"#0.1E5@70Ex"
5e (now that 0\K\#!0& so& 1@1'#\5@70Ex"\1@14
%hen& 1!E0.1x"@70EK"V#0.1E1@1'#7@45V0.15
!" is insufficient.
0nswer is C
45.
Gate # 0^!@+& the 4uestion as(s how shall 0 change to cope with a 100. increase of
+ to ma(e the rate constant.
x0"^!@!+" # 0^!@+ #V x^!#! #V x#1.414 #V 0 has to increase to 1.4140&
e4uivalent to sa, an increase of approximate 40.
46.
let ,#x& the number of people wor(ing more than 10 ,ears#560
560*x"@'00*x"#60.#Vx#!00
47.
!.0E100E1.0"@!#50E0@40
4".
1et the least one is x. 5hen other 10 populations have the greatest value& x will have
the minimum value.
KE10W1.1K#12!000
K#11000
0nswer is C
4%.
1et the price at the beginning is 1& then at the end of the first 4uarter& it was 1.!& at the
end of the second 4uarter& it was 1.5.
1.5*1.!"@1.!#!5.
0nswer is +
50.
150Eprofit"W40.#profit
$o& the profit is X100
51.
%he profit is sale price*cost
For one stoc(& sale price*cost"@cost#!0.& its cost is 66@1.!#'0
For the other stoc(& sale price*cost"@cost#*!0.& cost is 66@0.'#1!0
%hen& the total profit for the two stoc(s is;
66*'0E66*1!0#*'
0nswer is 7
5!.
60.W1!0.@40.E60.W1!0."#64.
52.
0m, was the 60th percentile of the '0 grades for her class& therefore& 10. are higher
than 0m,>s& 10.W'0#'.
16 of the other class was higher than 0m,. %otall,& 'E16#!7
%hen& the percentile is;
1'0*!7"@1'0#'5@100
0nswer is C
6or< ' =ate
1. 1et a be the number of hours it ta(es ?achine 0 to produce 1 widget on its own.
1et b be the number of hours it ta(es ?achine + to produce 1 widget on its own.
%he 4uestion tells us that ?achines 0 and + together can produce 1 widget in 2
hours. %herefore& in 1 hour& the two machines can produce 1@2 of a widget. 3n 1
hour& ?achine 0 can produce 1@a widgets and ?achine + can produce 1@b
widgets. %ogether in 1 hour& the, produce 1@a E 1@b # 1@2 widgets.
3f ?achine 0>s speed were doubled it would ta(e the two machines ! hours to
produce 1 widget. 5hen one doubles the speed& one cuts the amount of time it
ta(es in half. %herefore& the amount of time it would ta(e ?achine 0 to produce 1
widget would be a@!. Bnder these new conditions& in 1 hour ?achine 0 and +
could produce 1@a@!" E 1@b # 1@! widgets. 5e now have two un(nowns and two
different e4uations. 5e can solve for a.
%he two e4uations;
!@a E 1@b # 1@! Gemember& 1@a@!" # !@a"
1@a E 1@b # 1@2
$ubtract the bottom e4uation from the top;
!@a F 1@a # 1@! F 1@2
1@a # 2@6 F !@6
1@a # 1@6
%herefore& a # 6.
%he correct answer is ).
!.
+ecause 0dam and +rianna are wor(ing together& add their individual rates to find
their combined rate;
50 E 55 # 105 tiles per hour
%he 4uestion as(s how long it will ta(e them to set 1400 tiles.
%ime # 5or( @ Gate # 1400 tiles @ 105 tiles @ hour" # 40@2 hours # 12 and 1@2 hours #
12 hours and !0 minutes
%he correct answer is 7.
2.
%o find the combined rate of the two machines& add their individual rates;
25 copies@minute E 55 copies@minute # 60 copies@minute.
%he 4uestion as(s how man, copies the machines ma(e in half an hour& or 20
minutes.
60 copies@minute A 20 minutes # !&700 copies.
%he correct answer is +.
4. %om>s individual rate is 1 Hob @ 6 hours or 1@6.
Curing the hour that %om wor(s alone& he completes 1@6 of the Hob using rt # w".
Oeter>s individual rate is 1 Hob @ 2 hours.
Oeter Hoins %om and the, wor( together for another hourL Oeter and %om>s
respective individual rates can be added together to calculate their combined rate;
1@6 E 1@2 # 1@!.
5or(ing together then the, will complete 1@! of the Hob in the 1 hour the, wor(
together.
0t this point& !@2 of the Hob has been completed 1@6 b, Oeter alone E 1@! b, Oeter
and %om"& and 1@2 remains.
5hen John Hoins %om and Oeter& the new combined rate for all three is; 1@6 E 1@2
E 1@! # 1.
%he time that it will ta(e them to finish the remaining 1@2 of the Hob can be
solved;
rt # w 1"t" # 1@2 t # 1@2.
%he 4uestion as(s us for the fraction of the Hob that Oeter completed. 3n the hour
that Oeter wor(ed with %om he alone completed; rt # w w # 1@2"
1" # 1@2 of the Hob.
3n the last 1@2 of an hour that all three wor(ed together& Oeter alone completed;
1@2"1@2" # 1@6 of the Hob.
0dding these two values together& we get 1@2 E 1@6 of the Hob # 4@6 of the Hob.
%he correct answer is ).
5. 5e can solve this problem as a R37 Rariable 3n 0nswer 7hoice" and plug in
values for the two variables& " and y. 1et>s sa, " # ! and y # 2.
?achine 0 can complete one Hob in ! hours. %hus& the rate of ?achine 0 is 1@!.
?achine + can complete one Hob in 2 hours. %hus& the rate of ?achine + is 1@2.
%he combined rate for ?achine 0 and ?achine + wor(ing together is; 1@! E 1@2 #
5@6.
Bsing the e4uation Gate"%ime" # 5or(& we can plug 5@6 in for the combined rate&
plug 1 in for the total wor( since the, wor( together to complete 1 Hob"& and
calculate the total time as 6@5 hours.
%he 4uestion as(s us what fraction of the Hob machine + will -/% have to complete
because of 0>s help. 3n other words we need to (now what portion of the Hob machine
0 alone completes in that 6@5 hours.
0>s rate is 1@!& and it spends 6@5 hours wor(ing. +, plugging these into the G%#5
formula& we calculate that& 0 completes 1@!"6@5" # 2@5 of the Hob. %hus& machine +
is saved from having to complete 2@5 of the Hob.
3f we plug our values of " # ! and y # 2 into the answer choices& we see that onl,
answer choice ) ,ields the correct value of 2@5.
6. 5e can solve this problem as a R37 Rariable 3n answer 7hoice" and plug in
values for the variable ". 1etUs sa, " # 6. -ote that there is a logical restriction
here in terms of the value of ". 1indsa, has to have a rate of less than less than 1
room per hour if she needs JosephUs help to finish in an hour".
3f 1indsa, can paint 1@6 of the room in !0 minutes 1@2 of an hour"& her rate is 1@!.
rt # w
r1@2" # 1@6
r # 1@!
1et ; be the number of hours it ta(es Joseph to paint the entire room. JosephUs rate
then is 1@;. Joseph and 1indsa,Us combined rate is 1@< E 1@;& which can be
simplified;
1@! E 1@; ; @ !; E ! @ !; ; E !" @ !;
3f the two of them finish the room in one hour& using the formula of rt # w& we can
solve for ;.
rt . w and t . 1 hour"& w # 1 Hob"
; E !" @ !; "1" # 1 ; E ! # !; ; # !
%hat means that JosephUs rate is 1@!& the same as 1indsa,Us. %he 4uestion though as(s
us what fraction of the room Joseph would complete in !0 minutes& or 1@2 of an hour.
rt # w
1@!"1@2" # w
w # 1@6
-ow we must loo( at the answer choices to see which one is e4ual to 1@6 when we
plug in " # 6. /nl, 7 wor(s; 6 F 2" @ 1' # 1@6.
%he correct answer is 7.
7.
%he combined rate of individuals wor(ing together is e4ual to the sum of all the
individual wor(ing rates.
1et s # rate of a smurf& e # rate of an elf& and f # rate of a fair,. 0 rate is expressed in
terms of treehouses@hour. $o for instance& the first e4uation below sa,s that a smurf
and an elf wor(ing together can build 1 treehouse per ! hours& for a rate of 1@!
treehouse per hour.
s E e # 1@!
!" s E ! f # 1@!
e E f # 1@4
%he three e4uations can be combined b, solving the first one
for s in terms of e& and the third e4uation for f in terms of e& and
then b, substituting both new e4uations into the middle
e4uation.
1" s # 1@! F e
!" 1@! F e" E ! 1@4 F e" # 1@!
2" f # 1@4 F e
-ow& we simpl, solve e4uation ! for e;
1@! F e" E ! 1@4 F e" # 1@!
!@4 F e E !@4 F ! e # !@4
4@4 F 2e # !@4
*2e # *!@4
e # !@1!
e # 1@6
/nce we (now e& we can solve for s and f;
s # 1@! F e
s # 1@! F 1@6
s # 2@6 F 1@6
s # !@6
s # 1@2
f # 1@4 F e
f # 1@4 F 1@6
f # 2@1! F !@1!
f # 1@1!
5e add up their individual rates to get a combined rate;
e E s E f #
1@6 E 1@2 E 1@1! #
!@1! E 4@1! E 1@1! # 7@1!
Gemembering that a rate is expressed in terms of treehouses@hour& this indicates that a
smurf& an elf& and a fair,& wor(ing together& can produce 7 treehouses per 1! hours.
$ince we want to (now the number of hours per treehouse& we must ta(e the
reciprocal of the rate. %herefore we conclude that it ta(es them 1! hours per 7
treehouses& which is e4uivalent to 1!@7 of an hour per treehouse.
%he correct answer is C.
%.
Gate is defined as distance divided b, time.
%herefore;
%he G0%) of machine 0 #
%he G0%) of machine + #
%he 7/?+3-)C G0%) of machine 0 and machine + #
%his expression can be simplified b, eliminating the roots in the denominators as
follows;
%he 4uestion as(s us for the time& t& that it will ta(e both machines wor(ing together
to finish one Hob.
Bsing the combined rate above and a distance of ! Hob& we can solve for t as follows;
%he correct answer is choice 1.
%.
$ince this is a wor( rate problem& we>ll use the formula rate A time # wor'. $ince
we>ll be calculating times& we>ll use it in the form time # wor' @ rate.
First let -
0
e4ual the time it ta(es to paint the houses under the speedup scenario. -
0

will e4ual the sum of the following two values;
1. the time it ta(es to paint y houses at a rate of "
!. the time it ta(es to paint '0 F y" houses at a rate of 1.!5".
%hen let -
=
e4ual the time it ta(es to paint all '0 houses at the
stead, rate of ".
-
0
#
y
"
E
'0 F y"
1.!5"
-
0
#
1.!5y
1.!5"
E
'0 F y"
1.!5"
-
0
#
'0 E
0.!5y"
1.!5"
-
=
#
'0
"
%he desired ratio is
-
0
-
=
. %his e4uals -
0
times the reciprocal of -
=
.
0s a 4uic( chec(& note that if y # '0& meaning the, paint 011 the houses at rate "
before bringing in the extra help& then -
0
@-
=
# 1 as expected.
%he correct answer is +.
10. %here are several wa,s to achieve sufficienc, in solving this rate problem& so the
4uestion cannot be rephrased in a useful manner.
1" 3-$BFF373)-%; %his statement provides the difference between the number
of hot dogs consumed b, the third*place finisher letUs call this t" and the number
of hot dogs consumed b, the winner letUs call this w". 5e now (now that w # t E
!4& but this does not provide sufficient information to solve for w.
!" 3-$BFF373)-%; %he third*place finisher consumed one hot dog per 15
seconds. %o simplif, the units of measure in this problem& letUs restate this rate as
4 hot dogs per minute. $tatement !" tells us that the winner consumed ' hot dogs
per minute. %his does not provide sufficient information to solve for w.
1" 0-C !" $BFF373)-%; %he rate of consumption multiplied b, elapsed time
e4uals the number of hot dogs consumed. %his e4uation can be restated as time #
hot dogs@rate. +ecause the elapsed time is e4ual for both contestants& we can set
the hot dogs@rate for each contestant e4ual to one another;
w@' # t@4
w # !t
$ubstituting w F !4 for t ,ields
-
0
-
=
.
'0 E
0.!5y
1.!5"
>
"
'0
-
0
-
=
.
'0E0.!5
y
100
-
0
-
=
.
0.' E
0.00!5
y
w # !w F !4"
w # !w F 4'
4' # w
%he correct answer is 7.
11.
%his is a wor( problem. 5e can use the e4uation 5or( # Gate A %ime ? # 7 A -" to
relate the three variables 5or(& Gate& and %ime. %he 4uestion as(s us to find the
number of newspapers printed on $unda, morning. 5e can thin( of the 9number of
newspapers printed: as the 9wor( done: b, the printing press. $o& the 4uestion is
as(ing us to find the wor( done on $unda, morning& or ?
sunda,
.
%he printing press runs from 1;00 0? to 4;00 0? on $unda, morning& so -
sunda,
# 2
hours. $ince ?
sunda,
# 7
sunda,
A -
sunda,
& or in this case ?
sunda,
# 7
sunda,
A 2& (nowing the
rate of printing& 7
sunda,
& will allow us to calculate ?
sunda,
the number of newspapers
printed on $unda, morning". %herefore& the rephrased 4uestion becomes; 5hat is
7
sunda,
Q
1" 3-$BFF373)-%; %his statement tells us that 7
saturda,
# !7
sunda,
. 5hile this relates
$aturda,Us printing rate to $unda,Us printing rate& it gives no information about the
value of either rate.
!" 3-$BFF373)-%; For $aturda, morning& ?
saturda,
# 4&000 and -
saturda,
# 4 hours. 5e
can set up the following e4uation;
?
saturda,
# 7
saturda,
A -
saturda,
4&000 # 7
saturda,
A 4
7
saturda,
# 1&000
%his gives the rate of printing on $aturda, morning& but fails to give an, information
about $unda,Us rate.
1" 0-C !" $BFF373)-%; $tatement 1" tells us that 7
saturda,
# !7
sunda,
and statement
!" tells us that 7
saturda,
# 1&000. Outting this information together ,ields;
7
saturda,
# !7
sunda,
1&000 # !7
sunda,
500 # 7
sunda,
%he correct answer is 7.
!#.
%o find the combined rate of ?achines 0 and +& we combine their individual rates. 3f
?achine 0 can fill an order of widgets in a hours& then in 1 hour it can fill of the
order. +, the same to(en& if ?achine + can fill the order of widgets in b hours& then
in 1 hour& it can fill of the order. $o together in 1 hour& ?achines 0 and + can fill
of the order;
$o in 1 hour& ?achines 0 and + can complete of the order. %o find the number
of hours the machines need to complete the entire order& we can set up the following
e4uation;
fraction of order completed in 1 hour" x number of hours needed to complete entire
order" # 1 order.
3f we substitute for the fraction of the order completed in 1 hour& we get;
& where " is the number of hours needed to complete the entire order. 3f
we divide both sides b, & we get;
3n other words& it will ta(e ?achines 0 and + hours to complete the entire
order wor(ing together at their respective rates.
%he 4uestion stem tells us that a and b are both even integers. 5e are then as(ed
whether a and b are e4ual. 3f the, are e4ual& we can express each as !&& where & is a
non*zero integer& because the, are even. 3f we replace a and b with !& in the
combined rate& we get;
$o if a and b are e4ual& the combined rate of ?achines 0 and + must be an integer
since & is an integer". 5e can rephrase the 4uestion as;
3s the combined rate of ?achines 0 and + an integerQ
$tatement 1 tells us that it too( 4 hours and 4' minutes for the two machines to fill
the order remember& the, began at noon". %his shows that the combined rate of
?achines 0 and + is -/% an integer otherwise& it would have ta(en the machines a
whole number of hours to complete the order". $o we (now that a and b cannot be the
same. $ufficient.
$tatement ! tells us that . $ince both a and b must be positive because
the, represent a number of hours"& we can ta(e the s4uare root of both sides of the
e4uation without having to worr, about negative roots. %herefore& it must be true that
a E b # !0. $o it is possible that a # 10 and that b # 10& which would allow us to
answer D,esD to the 4uestion. +ut it is also possible that a # 1! and b # ' or an, other
combination of positive even integers that sum to !0"& which would give us a
DnoD. 3nsufficient.
%he correct answer is 0; $tatement 1 alone is sufficient& but statement ! alone is not.
12. 3f water is rushing into tan( 1 at " gallons per minute while lea(ing out at y
gallons per minute& the net rate of fill of tan( 1 is " F y. %o find the time it ta(es
to fill tan( 1& divide the capacit, of tan( 1 b, the rate of fill; & @ " F y".
5e (now that the rate of fill of tan( ! is y and that the total capacit, of tan( ! is twice
the number of gallons remaining in tan( 1 after one minute. 0fter one minute& there
are " F y gallons in tan( 1& since the net fill rate is " F y gallons per minute. %hus& the
total capacit, of tan( ! must be !" F y".
%he 4uestion as(s us if
tan( 1 fills up before tan( !.

$BFF373)-%; 5e can manipulate &y \ !"
!
@ 4"y E !y
!
;
%he time it ta(es to fill tan( two then is
!" F y"
y
.
5e can restate the 4uestion; 3s
&
" @ y
\
!" F y"
y
Q
&y \ !"
!
F 4"y E !y
!

&y \ !"
!
F !"y E y
!
"
&y \ !" F y"" F y" dividing b, " F y is o(a, since " F y V 0"
dividing b, y is o(a, since y V 0"
%his manipulation shows us that the time it ta(es to fill tan( 1 is definitel, longer than
the time it ta(es to fill tan( !.
3-$BFF373)-%; 5e can express this statement algebraicall, as; 1@!z" V
!" F y". 5e cannot use this expression to provide us meaningful information
about the 4uestion.
%he correct answer is 0.
!4.
From the 4uestion stem& we (now that +ill>s rate is 1 well per "= hours and 7arlos>s
rate is 1 well per y= hours.
%herefore& their combined rate is .
$ince 6 is the amount of time it ta(es +ill and 7arlos to dig a well together& we can
use the rate formula to find 6 in terms of " and y. 5e can rearrange the
formula to isolate -; .
$ince 6 is the amount of time -" it ta(es the two men to dig 1 well together& the
DdistanceD )" here is 1 well. %herefore& . $o we (now
that .
%he 4uestion then becomes; 3s an integerQ
$tatement 1" tells us that . 5e now (now that . 5e can
substitute for y and simplif,;
&y
" @ y
\
!
" F
y"
&
" @ y
\
!" F y"
y
3s this sufficient to tell us whether 6 is an integerQ 1et>s tr, some numbers. 3f " # 5&
then . +ut if " # !& then . $o in one case we get an integer& in
another case we get a fraction. $tatement 1" alone is insufficient to answer the
4uestion.
$tatement !" tells us that y is a nonprime even number. %his means y can be an,
even number other than !. 5e cannot tell from this whether 6 is an integer. For
example& if y # 4 and " # !& then . +ut if y # 4 and " # 5& then
. $o in one case we get a fraction& in another we get an integer.
$tatement !" alone is insufficient to answer the 4uestion.
3f we ta(e the statements together& we (now that and that y is an even
number greater than ! because we are dealing with rates here& we do not have to
worr, about zero or negative evens".
1et>s begin b, anal,zing the denominator of 6& the expression " E 1. $ince y is even
and & " must be odd. %herefore " E 1 must be even. 3f " E 1 is even& it must
be the product of ! and some integer call it &" that is less than ".
-ow let>s anal,ze the numerator of 6& the expression "A. $ince " is greater than y& it
must be greater than !. %his means "= will have both ! and & as factors remember& &
is less than "".
%herefore both the ! and & in " E 1 the denominator of 6" will cancel out with the !
and & in "= the numerator of 6"& leaving onl, the product of integers.
For example& if " # 5 and y # 4&

%herefore& if and if y is an even number greater than !& then 6 will alwa,s
be an integer.
%he correct answer is 7; +/%8 statements %/<)%8)G are sufficient but -)3%8)G
statement 01/-) is sufficient.
15.
1@4@1@4E1@'"#!@2
SPEE-
->ST+.CE
1.
1et b be the number of hours +ob spends bi(ing. %hen t F b" is the number of hours
he spends wal(ing. 1et d be the distance in miles from his home to school. $ince he
had the flat tire halfwa, to school& he bi(ed d@! miles and he wal(ed d@! miles. -ow
we can set up the e4uations using the formula rate x time # distance. Gemember that
we want to solve for d& the total distance from +ob>s home to school.
1" "b # d@!
!" yt F b" # d@!
$olving e4uation 1" for b gives us;
2" b # d@!" $ubstituting this value of b into e4uation ! gives;
4" yt F d@!"" # d@! ?ultipl, both sides b, !";
5" !"yt F d@!"" # d" Cistribute the !"y
6" !"yt F dy # d" 0dd dy to both sides to collect the d>s on one side.
7" !"yt # d" E dy Factor out the d
'" !"yt # d" E y" Civide both sides b, " E y" to solve for d
6" !"yt @ " E y" # d
%he correct answer is 7.
!. 5e begin b, figuring out 1ex,Us average speed. /n her wa, from 0 to +& she
travels 5 miles in one hour& so her speed is 5 miles per hour. /n her wa, bac(
from + to 0& she travels the same 5 miles at 15 miles per hour. 8er average speed
for the round trip is -/% simpl, the average of these two speeds. Gather& her
average speed must be computed using the formula 7- # )& where 7 is rate& - is
time and ) is distance. 8er average speed for the 54ole trip is the total distance
of her trip divided b, the total time of her trip.
5e alread, (now that she spends 1 hour going from 0 to +. 5hen she returns
from + to 0& 1ex, travels 5 miles at a rate of 15 miles per hour& so our formula
tells us that 15- # 5& or - # 1@2. 3n other words& it onl, ta(es 1ex, 1@2 of an hour&
or !0 minutes& to return from + to 0. 8er total distance traveled for the round trip
is 5E5#10 miles and her total time is 1E1@2#4@2 of an hour& or '0 minutes.
5e have to give our final answer in minutes& so it ma(es sense to find 1ex,>s
average rate in miles per minute& rather than miles per hour. 10 miles @ '0 minutes
# 1@' miles per minute. %his is 1ex,>s average rate.

5e are told that +en>s rate is half of 1ex,>s& so he must be traveling at 1@16 miles
per minute. 8e also travels a total of 10 miles& so 1@16"- # 10& or - # 160. +en>s
round trip ta(es 160 minutes.
0lternativel,& we could use a shortcut for the last part of this problem. 5e (now
that +en>s rate is half of 1ex,>s average rate. %his means that& for the entire trip&
+en will ta(e twice as long as 1ex, to travel the same distance. /nce we
determine that 1ex, will ta(e '0 minutes to complete the round trip& we can
double the figure to get +en>s time. '0 A ! # 160.
%he correct answer is C.
2. %here is an important (e, to answering this 4uestion correctl,; this is not a simple
average problem but a weighted average problem. 0 weighted average is one in
which the different parts to be averaged are not e4uall, balanced. /ne is Dworth
moreD than the other and s(ews the DsimpleD average in one direction. 3n
addition& we must note a unit change in this problem; we are given rates in miles
per hour but as(ed to solve for rates in miles per minute.
0verage rate uses the same C # G% formula we use for rate problems but we have to
figure out the different lengths of time it ta(es Can to run and swim along the total 4*
mile route. %hen we have to ta(e the 4 miles and divide b, that total time. First& Can
runs ! miles at the rate of 10 miles per hour. 10 miles per hour is e4uivalent to 1 mile
ever, 6 minutes& so Can ta(es 1! minutes to run the ! miles. -ext& Can swims !
miles at the rate of 6 miles per hour. 6 miles per hour is e4uivalent to 1 mile ever, 10
minutes& so Can ta(es !0 minutes to swim the two miles.
Can>s total time is 1! E !0 # 2! minutes. Can>s total distance is 4 miles. Cistance @
time # 4 miles @ 2! minutes # 1@' miles per minute.
-ote that if ,ou do not weight the averages but merel, ta(e a simple average& ,ou
will get !@15& which corresponds to incorrect answer choice +. 6 mph and 10 mph
average to 'mph. 'mph"1h@60min" # '@60 miles@minute or !@15 miles per minute.
%he correct answer is 0.
4. %he formula to calculate distance is Cistance # Gate"%ime". $o at an, given
moment %om>s distance let>s call it )
-
" can be expressed as )
-
# 6%. $o& at an,
given moment& 1inda>s distance let>s call it )
B
" can be expressed as )
B
# !- E 1"
remember& 1inda>s time is one hour more than %om>s". %he 4uestion as(s us to
find the positive difference between the amount of time it ta(es %om to cover half
of 1inda>s distance and the time it ta(es him to cover twice her distance. 1et>s
find each time separatel, first.
5hen %om has covered half of 1inda>s distance& the following e4uation will hold;
6- # !% E 1""@!. 5e can solve for -;
6- # !- E 1""@!
6- # !- E !"@!
6- # - E1
5- # 1
- # 1@5
$o it will ta(e %om 1@5 hours& or 1! minutes& to cover half of 1inda>s distance.
5hen %om has covered twice 1inda>s distance& the following e4uation will hold;
6- # !!- E 1". 5e can solve for -;
6- # !!- E 1"
6- # !!- E !"
6- # 4- E 4
!- # 4
- # !
$o it will ta(e %om ! hours& or 1!0 minutes& to cover twice 1inda>s distance.
5e need to find the positive difference between these times; 1!0 F 1! # 10'.
%he correct answer is ).
5. 0 4uestion with variables in the answer choices R37" can be solved b, pic(ing
values for the variables.
1et>s pic( the following values for x& , and z;
"
4
time for high speed travel
y
6
time regular travel
&
1!
distance from 0 to +
5hen pic(ing values for a R37 4uestion& it is best to pic( numbers that are eas, to
wor( with i.e.& 1! is divisible b, 4 and 6 here"& but that don>t have an, extraneous
relationships between them. For example " # 4& y # 2& & # 1! would be a less
favorable set of numbers because "y would e4ual & in that case and there is no need
for the product of the two times to e4ual the distance. Oic(ing variables with
extraneous relationships can lead to false positives when chec(ing the answer
choices.
-ow let>s solve the 4uestion according to the values we selected.
3f the high*speed train travels the 1! miles from 0 to + in 4 hours& it is traveling at 2
mph.
3f the regular train travels the 1! miles from 0 to + in 6 hours& it is traveling at ! mph.
%o evaluate how far each train travels when the, move toward each other starting at
opposite ends& let>s set up an G%C chart.
8igh*speed Gegular %otal
G 2 !
% t t
C
d 1! F d
1!
5e can set*up two e4uations with two un(nowns and solve.
2t # d
E" !t # 1! F d
*****************
5t # 1!& so t # !.4
3n the !.4 hours it ta(es for the two trains to meet&
the high speed train will have traveled 2!.4" # 7.! miles&
and the regular train will have traveled !!.4" # 4.' miles.
%herefore the high speed train will have traveled 7.! F 4.' # !.4 miles farther than the
regular train.
!.4 is our target number.
1et>s see which of the five answer choices give us !.4 when we plug in our values for
"& y and &;
Olug
Gesult
?atch %argetQ
0"
!.4
Pes
+"
*!.4
-o
7"
60
-o
1!6 F
4"
4 E 6
1!4 F
6"
4 E 6
1!4 E 6"
6 F 4
C"
*4.'
-o
)"
4.' -o
/nl, 0 matches the target.
%his 4uestion can also be solved algebraicall,.
$ince the trains traveled the & miles in " and y hours& their speeds can be represented
as &@" and &@y respectivel,.
5e can again use an G%C chart to evaluate how far each train travels when the,
move toward each other starting at opposite ends. 3nstead of using another variable d
here& let>s express the two distances in terms of their respective rates and times.
8igh*speed Gegular %otal
G &@" &@y
% t t
C
&t@" &t@y
&
$ince the two distances sum to the total when the two trains meet& we can set up the
following e4uation;
&t@" E &t@y # & divide both sides of the e4uation b, &
t@" E t@y # 1 multipl, both sides of the e4uation b, "y
ty E t" # "y factor out a t on the left side
t" E y" # "y divide both sides b, " E y
%o find how much further the high*speed train went in this time;
t # "y
" E y
46"4 F
6"
4 E 6
46"6 F
4"
4 E 6
rate
high
A time" F rate
reg
A time"
rate
high
F rate
reg
" A time

%he correct answer is 0.
6. %o determine +illUs average rate of movement& first recall that Gate A %ime #
Cistance. 5e are given that the moving wal(wa, is 200 feet long& so we need
onl, determine the time elapsed during +illUs Hourne, to determine his average
rate.
%here are two wa,s to find the time of +illUs Hourne,. First& we can brea( down
+illUs Hourne, into two legs; wal(ing and standing. 5hile wal(ing& +ill moves at
6 feet per second. +ecause the wal(wa, moves at 2 feet per second& +illUs foot
speed along the wal(wa, is 6 F 2 # 2 feet per second. %herefore& he covers the
1!0 feet between himself and the bottlenec( in 1!0 feet"@2 feet per second" # 40
seconds.
-ow& how far along is +ill when he stops wal(ingQ 5hile that 40 seconds
elapsed& the crowd would have moved 40 seconds"2 feet per second" # 1!0 feet.
+ecause the crowd alread, had a 1!0 foot head start& +ill catches up to them at
1!0 E 1!0 # !40 feet. %he final 60 feet are covered at the rate of the moving
wal(wa,& 2 feet per second& and therefore re4uire 60 feet"@2 feet per second" #
!0 seconds. %he total Hourne, re4uires 40 E !0 # 60 seconds& and +illUs rate of
movement is 200 feet"@60 seconds" # 5 feet per second.
%his problem ma, also be solved with a shortcut. 7onsider that +illUs Hourne, will
end when the crowd reaches the end of the wal(wa, as long as he catches up
with the crowd before the wal(wa, ends". 5hen he steps on the wal(wa,& the
crowd is 1'0 feet from the end. %he wal(wa, travels this distance in 1'0 feet"@2
feet per second" # 60 seconds& and +illUs average rate of movement is 200 feet"@
60 seconds" # 5 feet per second.
%he correct answer is ).

z
"
F &
y
"
A "y
" E y
&y @ &"
"y
A "y
" E y
&y F ""
" E y
7. 3t is easier to brea( this motion up into different segments. 1et>s first consider the
40 minutes up until John stops to fix his flat.
40 minutes is !@2 of an hour.
3n !@2 of an hour& John traveled 15 A !@2 # 10 miles rt # d"
3n that same !@2 of an hour& Jacob traveled 1! A !@2 # ' miles
John therefore had a two*mile lead when he stopped to fix his tire.
3t too( John 1 hour to fix his tire& during which time Jacob traveled 1! miles.
$ince John began this 1*hour period ! miles ahead& at the end of the period he is
1! F ! # 10 miles behind Jacob.
%he 4uestion now becomes Dhow long does it ta(e John to bridge the 10*mile gap
between him and Jacob& plus whatever additional distance Jacob has covered&
while traveling at 15 miles per hour while Jacob is traveling at 1! miles per
hourQD 5e can set up an rt # d chart to solve this.
John Jacob
7 15 1!
- t t
) d E 10 d
John>s travel during this Dcatch*up periodD can be represented as 15t # d E 10
Jacob>s travel during this Dcatch*up periodD can be represented as 1!t # d
3f we solve these two simultaneous e4uations& we get;
15t # 1!t E 10
2t # 10
t # 2 1@2 hours
0nother wa, to approach this 4uestion is to note that when John begins to ride again&
Jacob is 10 miles ahead. $o John must ma(e up those first 10 miles plus whatever
additional distance Jacob has covered while both are riding. $ince Jacob>s additional
distance at an, given moment is 1!t measuring from the moment when John begins
riding again" we can represent the distance that John has to ma(e up as 1!t E 10. 5e
can also represent John>s distance at an, given moment as 15t. %herefore& 15t # 1!t E
10& when John catches up to Jacob. 5e can solve this 4uestion as outlined above.
%he correct answer is +.
".
Bse $& G and + to represent the individual race times of $tephanie& Gegine& and
+rian respectivel,. %he problem tells us that $tephanie and Gegine>s combined times
exceed +rian>s time b, ! hours. %herefore;
3n order to win the race& an individual>s time must be less than one*third of the the
combined times of all the runners. %hus& in order for +rian to win the race meaning
that +rain would have the lowest time"& his time would need to be less than one*third
of the combined times for all the runners. %his can be expressed as follows;
%his ine4ualit, can be simplified as follows;
Bsing the fact that & the ine4ualit, can be simplified even further;
%his tells us that in order for +rian to win the race& his time must be less than !
hours. 8owever& this is impossible= 5e (now that the fastest +rian runs is ' miles
per hour& which means that the shortest amount of time in which he could complete
the !0 mile race is !.5 hours.
%his leaves us with $tephanie and Gegine as possible winners. $ince the problem gives us
identical information about $tephanie and Gegine& we cannot eliminate either one as a
possible winner. %hus& the correct answer is C; $tephanie or Gegine could have won the
race
6. /ne wa, to approach this problem is to pic( numbers for the variables. $o let>s
sa, that
" . 60 miles per hour
y . 20 miles per hour
/n the initial trip& the car traveled for 6 hours at 60 miles per hour. $ince distance #
rate A time& the distance for this initial trip is 60 x 6 # 260 miles. %he return trip went
along the same 260*mile route& but at onl, 20 miles per hour. %his means that for the
return trip& 260 # 20 x time& so the duration of the return trip was 260@20 # 1! hours.
%he entire trip too( 6 E 1! # 1' hours which is e4ual to 1'60" minutes.
Olug our chosen values for " and y 60 and 20 respectivel," into the answer choices
and see which one ,ields the value 1'60". %he onl, one that does this is answer
choice 0;
.
0lternativel,& we can solve this problem using onl, algebra. 1et us call t the time in
hours for the return trip. %hen& using the formula distance # rate A time& we can sa,
that
distance for initial trip # " A 6& and distance for return trip # t A y.
$ince the distance for the initial trip e4uals the distance for the return trip& we can
combine the two e4uations to sa,
6" # ty
$olving for t& we get
%he total time for the round trip will be the time for the initial trip 6 hours" plus the
time for the return trip. )xpressed in minutes& this is

6 E
660
"
C0
" 60# 1'60"

6"
y
"
%he correct answer is 0.
!0.
%his standard rate problem will rel, heavil, on the formula 7-#)& where 7 is the
rate& - is the time and ) is the distance traveled.
First& we should find the driving and bi(ing distances;
3f Ceb drives for 45 minutes& or 0.75 hours& at a rate of 40mph& she drives a total
distance of
0.75"40" # 20 miles.
3f the bi(e route is !0. shorter than the driving route& the bi(e route is 20 F 200.!" #
20 F 6 # !4 miles.
-ext& we need to determine how long it will ta(e Ceb to travel the route b, bi(e. $he
wants to ensure that she>ll get to wor( b, a particular time& so we want to calculate the
longest possible time it could ta(e herL therefore& we have to assume she will bi(e at
the slowest end of the range of the speeds given; 1!mph. 3f she travels !4 miles at
1!mph& it will ta(e her !4@1! # ! hours or 1!0 minutes.
3f Ceb normall, ta(es 45 minutes to drive to wor( but could ta(e up to 1!0 minutes to
bi(e to wor(& then she must leave 1!0 F 45 # 75 minutes earlier than she normall,
does to ensure that she will arrive at wor( at the same time.
%he correct answer is C.
!!.
3f we want +renda>s distance to be twice as great as 0lex>s distance& we can set up the
following e4uation;
!4-" # 7- F 1"& where 4- is 0lex>s distance rate A time" and 7- F 1" is +renda>s
distance since +renda has been traveling for one hour less".
3f we simplif, this e4uation to isolate the % which represents 0lex>s total time"& we
get;
!4-" # 7- F 1"
'% # 7- F 7

6

E
6"
y
"
6
0
7 # 7- F '-
7 # -7 F '"
%his is choice 7.
!#.
%he (e, to solving this 4uestion lies in understanding the mathematical relationship
that exists between the speed s"& the circumference of the tires c" and the number of
revolutions the tires ma(e per second r". 3t ma(es sense that if ,ou (eep the speed of
the car the same but increase the size of the tires& the number of revolutions that the
new tires ma(e per second should go down. 5hat& however& is the exact
relationshipQ
$ometimes the best wa, to come up with a formula expressing the relationship
between different variables is to anal,ze the labels or units" that are associated with
those variables. 1etUs use the following units for the variables in this 4uestion even
though the, are slightl, different in the 4uestion";
c; inches@revolution
s; inches@sec
r; revolutions@sec
%he labels suggest; rev@sec" x inches@rev" # inches@sec"& which means that rc # s.
5hen the speed is held constant& as it is in this 4uestion& the relationship rc # s
becomes rc # '. r and c are inversel, proportional to one another. 5hen two
variables are inversel, proportional& it means that whatever factor ,ou multipl, one of
the variables b,& the other one changes b, the inverse of that factor. For example if
,ou (eep the speed constant and ,ou double the circumference of the tires& the rev@sec
will be halved. 3n this wa, the product of c and r is (ept constant.
3n this 4uestion the circumferences of the tires are given in inches& the speed in miles
per hour& and the rotational speed in revolutions per second. 8owever& the
discrepancies here donUt affect the fundamental mathematical relationship of inverse
proportionalit,; if the speed is (ept constant& the rev@sec of the tires will change in an
inverse manner to the circumference of the tires.
1etUs assign c
1
# initial circumferenceL c
!
# new circumference
r
1
# initial rev@sec L r
!
# new rev@sec
$ince the speeds are held constant;
c
1
r
1
# c
!
r
!
r
!
# c
1
@c
!
"r
1
r
!
# !'@2!"r
1

r
!
# 7@'"r
1
7
7 F '
# -
3f the new rev@sec is 7@' of the previous rev@sec& this represents a 1@' or 1!.5.
decrease and the correct answer is +". Gelationships of inverse proportionalit, are
important in an, word problem on the <?0% involving a formula in the form of "y #
& and in which & is held constant.
!(.
%he crux of this problem is recalling the average speed formula;
3n this particular case& since ?artha drove at one speed for some time and at another
speed for the remainder of the trip& the total time will be the sum of the times spent at
the two speeds. 1et be the time spent traveling at the first speed and let be the
time spent traveling at the second speed. ?artha>s average speed can then be
expressed as;
$ince we do not (now the total distance& we can call it d. 5e do not (now either
or & but we can express them in terms of d b, recalling that & where ) is the
distance and 7 is the rate.
1et>s find first. $ince ?artha traveled the first " percent of the Hourne, at 60 miles
per hour& ) for that portion of the trip will be e4ual to and will therefore be
e4ual to .
-ow let>s find . %he remaining distance in ?artha>s trip can be expressed as
. %herefore& will be e4ual to . 5e can plug these into our
average rate formula and simplif,;
5e cannot reduce this fraction an, further. %herefore& the numerator of ?artha>s
average speed is 20&000.
%he correct answer is ).
!4.
/ne 4uic( wa, to solve this problem is to create a chart relating each cloc( to the
next;

-otice that each
cloc( runs
relative to the
previous cloc(.
For example& when 7loc( Z! gains 15 minutes an hour& it does so for onl, 4.5 hours&
Geal %ime; 6 O?
7loc( Z1
Cispla,s;
6 O? F 15 min 6" # 6 O? F 60 min # 4;20 O?
7loc( Z!
Cispla,s;
4;20 O? E 15 min 4.5" # 4;20 O? E 67.5 min #
5;275 O?
7loc( Z2
Cispla,s;
5;275 O? F !0 min 5 5@'" # 5;275 O? F 11!.5 min
# 2;45 O?
7loc( Z4
Cispla,s;
2;45 O? E !0 min 2 2@4" # 2;45 O? E 75 min #
5;00 O?
since 7loc( Z1 progressed onl, 4.5 hours.
%he correct answer is 0; 0t 6 O? real time& 7loc( Z4 displa,s 5;00 O?
15.
%he hands of the cloc( will be perpendicular when the angle of the hour hand minus
the angle of the minute hand both relative cloc(wise to the ver, top of the cloc(& or
the 91!: position" is exactl, 60 degrees.
0t 7;00 exactl,& the minute hand is exactl, at the 91!: position& so it is at 0 degrees.
0 cloc( face is 260 degrees around and there are 60 minutes in an hour so each
minute elapsed will result in the minute hand moving 260@60 # 6 degrees cloc(wise.
%herefore& at " minutes past 7;00& the minute hand is at 6" degrees.
degrees. 3f the hour hand moves 20 degrees during the course of an hour& it
moves 1@! a degree ever, minute since there are 60 minutes in an hour". %herefore&
at " minutes past 7;00& the hour hand will be at !10 E 1@!" degrees.
5e want to solve for " which is the number of minutes past 7;00" such that the
following holds true align#centerVangle of hour hand" F angle of minute hand" #
60 degrees
%his can be rewritten mathematicall, as follows;
%he exact time that the hour and minutes hands are perpendicular is !1 6@11 minutes
past 7;00
!6.
5e (now that %eam 0 wins the race b, 7 seconds& which means that Gunner 4 on
%eam + will cross the finish line 7 seconds after Gunner 4 on %eam 0 crosses the
finish line. %hus& the 4uestion can be rephrased as follows; 8ow far does Gunner 4
on %eam + run in 7 secondsQ $ince his lap time is 4! seconds& he covers 7@4!& or 1@6&
of the trac( in 7 seconds.
%herefore& we must determine the length of the trac(. %he trac( is formed b, a
rectangle with two adHoining semicircles. %he length of the trac( is e4ual to ! times
the length of the rectangle plus the circumference of the circle the two semi*circles
combined".
%he diameter of the circle is; 1'0 meters F 1!0 meters # 60 meters. %hus& the radius
of the circle is 20 meters and the circumference is !r # 60 meters. Finall,& the
length of the trac( is; ! A 1!0 E 60" meters # !40 E 60" meters.
Gemember& Gunner 4 on %eam + still has 1@6 of the lap to run when Gunner 4 on
%eam 0 finishes the race. $o& %eam + loses the race b,; !40 E 60" @ 6 # 40 E
10" meters.
%he correct answer is +.
17.
Cistance # Gate A %ime& or ) # 7-.
1" 3-$BFF373)-%; %his statement tells us 8arr,Us rate& 20 mph. %his is not enough
to calculate the distance from his home to his office& since we donUt (now an,thing
about the time re4uired for his commute.
) # 7- # 20 mph" -"
) cannot be calculated because - is un(nown.
!" 3-$BFF373)-%; 3f 8arr, had traveled twice as fast& he would have gotten to
wor( in half the time& which according to this statement would have saved him 15
minutes. %herefore& his actual commute too( 20 minutes. $o we learn his commute
time from this statement& but donUt (now an,thing about his actual speed.
) # 7- # 7" 1@! hour"
) cannot be calculated because 7 is un(nown.
1" 0-C !" $BFF373)-%; From statement 1" we learned that 8arr,Us rate was 20
mph. From $tatement !" we learned that 8arr,Us commute time was 20 minutes.
%herefore& we can use the rate formula to determine the distance 8arr, traveled.
) # 7- # 20 mph" 1@! hour" # 15 miles
%he correct answer is 7.
1'.
%his 4uestion cannot necessaril, be rephrased& but it is important to recognize that we
need not necessaril, calculate 5end,Us or +obUs travel time individuall,.
Cetermining the difference between 5end,Us and +obUs total travel times would be
sufficient. %his difference might be expressed as t
b
F t
w
.
1" 3-$BFF373)-%; 7alculating +obUs rate of speed for an, leg of the trip will not
give us sufficient information to determine the time or distance of his Hourne,& at least
one of which would be necessar, to determine how 4uic(l, 5end, reaches the
restaurant.
!" $BFF373)-%; %o see wh, this statement is sufficient& it is helpful to thin( of
+ob>s Hourne, in two legs; the first leg wal(ing together with 5end, t
1
"& and the
second wal(ing alone t
!
". +ob>s total travel time t
b
# t
1
E t
!
. +ecause 5end, traveled
halfwa, to the restaurant with +ob& her total travel time t
w
# !t
1
. $ubstituting these
expressions for t
b
F t
w
&
t
1
E t
!
F !t
1
# t
!
F t
1

t
b
F t
w
# t
!
F t
1

$tatement !" tells us that +ob spent 2! more minutes traveling alone than with
5end,. 3n other words& t
!
F t
1
# 2!. 5end, waited at the restaurant for 2! minutes for
+ob to arrive.
%he correct answer is +.
16. %o determine the average speed for the trip from %ownsend to $mallville and
bac( again& we need to (now the average speed in each direction. +ecause the
distance in each direction is the same& if we have the average speed in each direction
we will be able to find the average speed of the entire trip b, ta(ing the total distance
and dividing it b, the total time.
$BFF373)-%; %his allows us to figure out the average speed for the return
trip. 3f the return time was 2@! the outgoing time& the return speed must have
been !@2 that of the outgoing. 5henever the distance is fixed& the ratio of the
times will be the inverse of the ratio of the speeds.
5e can see this b, loo(ing at an example. 1et>s sa, the distance between the two
towns was '0 miles.
<oing Geturning
G 40
%
C '0 '0
5e can calculate the DgoingD time as ! hours. $ince& the return trip too( 50. longer&
the Dreturning timeD is 2 hours. %hus& the average rate for the return trip is
Cistance@%ime or '0@2 miles per hour.
<oing Geturning
G 40 "0'(
% ! 2
C '0 '0
5e can use this table to calculate the average speed for the entire trip; ta(e the total
distance& 160& and divide b, the total time& 5.
<oing Geturning %/%01
G 40 "0'( ???
% ! 2 5
C '0 '0 160
%his results in an average speed of 2! miles per hour.
3t does not matter that we chose a random distance of '0L we would able to solve
using an, distance or even using a variable " as the distance. %he times would adHust
accordingl, based on the distance we used and the same average speed of 2! would
result.
3-$BFF373)-%; 3f all we (now is the distance from Giverdale to $mallville&
we will be able to find the time traveled on the wa, there but we will have no
indication of how fast the car traveled on the wa, bac( and therefore no wa,
of (nowing what the average overall speed was.
%he correct answer is 0.
!0.
%he average speed is defined to be the total distance of the trip& divided b, the total
time of the trip. %he 4uestion stem tells us the distance from -ew Por( to +oston is
!50 miles& so we can rephrase the 4uestion as D8ow long did it ta(e +ill to drive from
-ew Por( to +ostonQD
$BFF373)-%; $tatement 1" tells us it too( +ill 5 hours to drive from -ew
Por( to +oston& answering the rephrased 4uestion. 3n fact& his average rate of
speed e4uals !50@5 # 50 miles per hour.
3-$BFF373)-%; $tatement !" tells us that at the midpoint of the trip +ill
was going exactl, 50 miles per hour& but we can>t figure out how long the trip
too( from this information. +ill may have traveled at a constant rate of 50
mph throughout the whole trip& but he might also have been going faster or
slower at different times.
%he correct answer is 0.
!1.
5e can attac( this problem b, first setting up a rate chart& identif,ing what we
alread, (now& and using variables for an, un(nown values;
%rain 0 %rain +
Gate 100 mph r
%ime ! * t t
Cistance d d
%he chart ,ields two e4uations;
5e also (now that when the trains pass each other going in opposite directions"&
%rain 0 has been traveling for 1 hour and %rain + has been traveling for 10 minutes
or 1@6 of an hour.
%his means that %rain 0 has traveled 100 miles and %rain + has traveled miles.
%hus& the total distance from -ew Por( to +oston can be expressed using the
e4uation& .
5e now have three e4uations and three variables;
$etting e4uation 0" and e4uation +" to be e4ual& we can solve for t& as follows;
%hen we can set e4uation +" and e4uation 7" to be e4ual& substitute for t& and solve
for r as follows;
%hus& the rate& r& of %rain + is either 200 mph or !00 mph. Bsing this information we
can chart out the two possible scenarios.
S&enario !@ Train 1 4as a rate of (00 mp4. 3t travels 50 miles in 1@6 hour& at which
point it meets %rain 0 which has alread, traveled 100 miles. %herefore& the total
distance from +oston to -ew Por( must be 150 miles. %hus& %rain +>s total traveling
time was 1@! hour& and %rain 0>s total traveling time was 1 1@! hours. %rain + arrived
in -ew Por( at 4;!0 O? and %rain 0 arrived in +oston at 4;20 O?.
S&enario #@ Train 1 4as a rate of #00 mp4. 3t travels 22 1@2 miles in 1@6 hour& at
which point it meets %rain 0 which has alread, traveled 100 miles. %herefore& the
total distance from +oston to -ew Por( must be 122 1@2 miles. %hus %rain +>s total
traveling time was !@2 hour& and %rain 0>s total traveling time was 1 1@2 hours. %rain
+ arrived in -ew Por( at 4;20 O? and %rain 0 arrived in +oston at 4;!0 O?.
$tatement 1" tells us that %rain + arrived in -ew Por( before %rain 0 arrived in
+oston. From this& we (now that S&enario ! must have occurred and %rain + arrived
in -ew Por( at 4;!0 O?. 5e have sufficient information to answer the 4uestion.
$tatement !" tells us that the distance between -ew Por( and +oston is greater than
140 miles. %his means that S&enario # is not possible so S&enario ! must have
occurred; %rain + arrived in -ew Por( at 4;!0 O?. 0gain& we have sufficient
information
to answer the 4uestion.
%he correct answer is C"; )ach statement 01/-) is sufficient.
!!. %he 4uestion as(s for the percent decrease in )dwinUs travel time. %o determine
this& we need to be able to find the ratio between& %
1
the travel time if )dwin
drives alone" and %
!
the travel time if )dwin and <eorge drive together". -ote
that we do -/% need to determine specific values for %
1
and %
!
L we onl, need to
find the ratio between them.
5h,Q Oercentage change is defined as follows;
Bltimatel,& we can solve the percentage change e4uation above b, simpl,
determining the value of .
Bsing the formula Gate A %ime # Cistance& we can write e4uations for each of the !
possible trips.
%
1
# %ravel time if )dwin drives alone
%
!
# %ravel time if )dwin and <eorge drive together
) # )dwinUs Gate
< # <eorgeUs Gate
C # Cistance of the trip
3f )dwin travels alone;
3f )dwin and <eorge travel together;
$ince )dwin and <eorge split the driving e4uall,& the rate for the trip is e4ual to the
average of )dwin and <eorgeUs individual rates".
$ince both trips cover the same distance C"& we can combine the ! e4uations as
follows;
%hen& we can isolate the ratio of the times %
!
@%
1
" as follows;
-ow we loo( at the statements to see if the, can help us to solve for the ratio of the
times.
$tatement 1" gives us a value for C& the distance& which does not help us since C is
not a variable in the ratio e4uation above.
$tatement !" tells us that <eorgeUs rate is 1.5 times )dwinUs rate. %hus& < # 1.5).
5e can substitute this information into the ratio e4uation above;
%hus& using this ratio we can see that )dwinUs travel time for the trip will be reduced
as follows;
$tatement !" alone is sufficient to answer the 4uestion. %he correct answer is +.
!2. 5e are as(ed to find the time that it ta(es %rain + to travel the entire distance
between the two towns.
$BFF373)-%; %his tells us that + started traveling 1 hour after %rain 0
started traveling. From the 4uestion we (now that %rain 0 had been traveling
for ! hours when the trains passed each other. %hus& train +& which started 1
hour later& must have been traveling for ! * 1 # 1 hour when the trains passed
each other.
1etUs call the point at which the two trains pass each other Ooint O. %rain 0 travels
from %own 8 to Ooint O in ! hours& while %rain + travels from %own < to Ooint O in
1 hour. 0dding up these distances and times& we have it that the two trains covered
the entire distance between the towns in 2 i.e. ! E 1" hours of combined travel time.
$ince both trains travel at the same rate& it will ta(e 2 hours for either train to cover
the entire distance alone. %hus& from $tatement 1" we (now that it will ta(e %rain +
2 hours to travel between %own < and %own 8.
3-$BFF373)-%; %his provides the rate for %rain +. $ince both trains travel at
the same rate& this is also the rate for %rain 0. 8owever& we have no
information about when %rain + started traveling relative to when %rain 0
started traveling" and we have no information about the distance between
%own < and %own 8. %hus& we cannot calculate an, information about time.
%he correct answer is 0.
#4.
$ince 0+ # +7& triangle 0+7 is a 45*45*60 triangle. $uch triangles have fixed side
ratios as follows;
%hus& we can call <reg>s distance 0+" x& while +rian>s distance 07" is
. +rian has a greater distance to travel.
0etAs first analBCe Statement (! alone@ Dre8As a9era8e speed is t4at of
1rianAs.
%his indicates that +rian is traveling 1.5 times faster than <reg. 3f <reg>s rate is r&
than +rian>s rate is 1.5r. 8owever& recall that +rian also has a greater distance to
travel.
%o determine who will arrive first& we use the distance formula;
5hoever has a shorter T>*E will arrive first.
<reg>s time
#
+rian>s time #
$ince +rian is traveling for less time& he will arrive first. $tatement 1" alone is
sufficient.
0etAs no5 analBCe Statement (# alone@ 1rianAs a9era8e speed is #0 miles per
4o2r 8reater t4an Dre8As.
%his gives us no information about the ratio of +rain>s average speed to <reg>s
average speed. %hus& although we (now that +rian>s distance is approximatel, 1.4
times <reg>s speed& we do not (now the ratio of their speeds& so we can not
determine who will arrive first.
For example& if +rian travels at !5 mph& <reg travels at 5 mph. 3n this case +rian
arrives first. 8owever& if +rian travels at 100 mph& <reg travels at '0 mph. 3n this
case <reg arrives first.
%herefore& $tatement !" alone is not sufficient.
$ince statement 1" alone is sufficient& but statement !" alone is not sufficient& the
correct answer is 0.
#5.
From 1& RV16feet@second#16@5!'0"@1@2600"#10.6mile@hour& the distance that he
c,cled is greater than 10.6W1@!#5.45. 5e cannot (now whether it greater than 6.
From !& R\1'feel@second#1'@5!'0"@1@2600"#1!.!7mile@hour& the distance that he
c,cled is less than 1!.!7W1@!#6.125.
7ombine 1 and !& 5.45\the distance\6.125& insufficient.
0nswer is ).
#6.
1". 3nsufficient& no idea about the car>s speed for the next !00 (ms
!" 0ctual speed E !0" W actual time * 1 hour" # 400 (ms # actual speed W actual time
7an solve for actual time
0nswer is +
#7.
1". x@50E520*x"@60#10& so& x can be solve out and x@50 will be the answer
!". %he time cost on two distances could be 5h& 1hL 6h& !hL... insufficient.
0nswer is 0
#".
5hen c,clist stops& the distance between he and the hi(er is !0W5*4W5'0
$o& the c,clist will wait '0@4!0 minutes.
!6.
60@vE2" * 60@v*2"#0.5
v#22
$o& t#60@26#!.5
S> ' C> ' Pop2lation Dro5t4
1.
%he investment contract guarantees to ma(e three interest pa,ments;
X10&000 initial investment"
E X!00 1. interest on X10&000 principal # X100& so !. # ! A X100"
X10&!00
E X206 1. interest on X10&!00 principal # X10!& so 2. # 2 A X10!"
X10&506
E X4!0.!4 1. interest on X10&506 principal # X105.06& so 4. # 4 A X105.06"
X10&6!6.!4
%he final value is X10&6!6.!4 after an initial investment of X10&000. %hus& the total
amount of interest paid is X6!6.!4 the difference between the final value and the
amount invested".
%he correct answer is ).
!.
3f we decide to find a constant multiple b, the hour& then we can sa, that the
population was multiplied b, a certain number three times from 1 p.m. to 4 p.m.;
once from 1 to ! p.m.& again from ! to 2 p.m.& and finall, from 2 to 4 p.m.
1et>s call the constant multiple ".
!&000"""""" # !50&000
!&000"
2
" # !50&000
"
2
# !50&000@!&000 # 1!5
" # 5
%herefore& the population gets five times bigger each hour.
0t 2 p.m.& there were !&0005"5" # 50&000 bacteria.
%he correct answer is 0.
2.
0 population problem on the <?0% is best solved with a population chart that
illustrates the swarm population at each unit of time. 0n example of a population
chart is shown below;
%ime Oopulation
4 hours ago 1&000
! hours ago !&000
-/5 4&000
in ! hours '&000
in 4 hours 16&000
in 6 hours 2!&000
in ' hours 64&000
in 10 hours 1!'&000
in 1! hours !56&000
0s can be seen from the chart& in 1! hours the swarm population will be e4ual to
!56&000 locusts. %hus& we can infer that the number of locusts will exceed !50&000 in
slightl, less than 1! hours. $ince we are as(ed for an approximate value& 1! hours
provides a sufficientl, close approximation and is therefore the correct answer.
%he correct answer is C
4. 5e need to consider the formula for compound interest for this problem; $ . #1
E r4
"
, where $ is the final value of the investment& # is the principal& r is the
interest rate per compounding period as a decimal& and " is the number of
compounding periods -/%); sometimes the formula is written in terms of the
annual interest rate& the number of compounding periods per ,ear and the number
of ,ears". 1et>s start b, manipulating the given expression for r;
1etUs compare this simplified e4uation to the compound interest formula. -otice that
r in this simplified e4uation and in the 4uestion" is not the same as the r in the
compound interest formula. 3n the formula& the r is alread, expressed as a decimal
e4uivalent of a percent& in the 4uestion the interest is r percent. %he simplified
e4uation& however& deals with this discrepanc, b, dividing r b, 100.
3n our simplified e4uation& the cost of the share of stoc( p"& corresponds to the
principal #" in the formula& and the final share price v" corresponds to the final
value $" in the formula. -otice also that the exponent ! corresponds to the " in the
formula& which is the number of compounding periods. +, comparing the simplified
e4uation to the compound interest formula& we see that the e4uation tells us that the
share rose at the dail, interest rate of p percent for %5/ da,s. %hen the share lost a
value of 6 dollars on the third da,& i.e. the 9F 6: portion of the expression. 3f the
investor bought the share on ?onda,& she sold it three da,s later on %hursda,.
%he correct answer is +.
5.
7ompound interest is computed using the following formula;
$ # # 1 E r@n"
nt
& where
$ # Final value
# # Orincipal
r # annual interest rate
n # number of compounding periods per ,ear
t # number of ,ears
From the 4uestion& we can deduce the following information about the growth during
this period;
0t the end of the " ,ears& the final value& $& will be e4ual to 16 times the principal
the mone, is growing b, a factor of 16".
%herefore& $ # 16#.
r # .0' '. annual interest rate"
n # 4 compounded 4uarterl,"
t # " the 4uestion is as(ing us to express the time in terms of " number of ,ears"
5e can write the e4uation
16# # # 1 E .0'@4"
4"

16 # 1.0!"
4"
-ow we can ta(e the fourth root of both sides of the e4uation. i.e.the e4uivalent of
ta(ing the s4uare root twice" 5e will onl, consider the positive root because a
negative ! doesn>t ma(e sense here.
16
1@4
# S1.0!"
4"
T
1@4
! # 1.0!"
"
%he correct answer is +.
6.
0lthough this problem appears to be complicated& it is fairl, straightforwardL since
we are given a formula& we can simpl, plug in the values that we need then calculate.
First& let us assign a value to each of the variables in the formula;
B # amount of the loan # 1000 F 7 # 662
r # annual interest rate # 10. # 0.1
: # compounding factor # 1 E r"
%
# 1.1"
2
# 1.1"1.1"1.1" # 1.!11.1" # 1.221
8ence O # 662 x 1.221 x 0.1" @ 1.221 F 1" # 662 x 1.221 x 0.1" @ 0.221 # 662@.221"
x 1.221 x 0.1.
-ote that 662 is an integral multiple of 0.221 and 662@0.221 # 662000@221 # 2000.
8ence O # 662@0.221" x 1.221 x 0.1 # 2000 x 1.221 x 0.1 # 266.20
%he correct answer is C.
-ote; %he <?0% will never as( ,ou to explicitl, divide b, a 2*digit number unless
there is some wa, to simplif, the division or there is a tric(& e.g.& dividing b,
a power of 5 such 1!5& or dividing b, something simple such as 111". 3f ,ou find that
,ou need to divide something li(e 467.26 b, 0.221 with no obvious wa, to simplif,&
,ou have most li(el, either missed an opportunit, to simplif, in a previous step& or
,ou have made a mista(e.
7.
%he 4uestion as(s us to find the monthl, pa,ment on a X1000 loan at 10. monthl,
interest compounded monthl, for three months. 1et>s define the following variables;
# # Orincipal # X1000
i # monthl, interest rate # 10. # 0.1
c # compound growth rate # 1 E i # 1.1
" # monthl, pa,ment to be calculated"
0t the start& 1ouie>s outstanding balance is #. Curing the next month& the balance
grows b, a factor of c as it accumulates interest& then decreases b, " when 1ouie
ma(es his monthl, pa,ment. %herefore the balance after month 1 is #c * ". )ach
month& ,ou must multipl, the previous balance b, c to accumulate the interest& and
then subtract " to account for 1ouie>s monthl, pa,ment. 3n chart form;
+alance at start; #
+alance after month 1; #c F "
+alance after month !; S#c F "Tc F " # #c
!
F "cE1"
+alance after month 2; S#c
!
* "cE1"Tc * " # #c
2
* "c
!
EcE1"
Finall,& the loan should be paid off after the third month& so the last loan balance must
e4ual 0. %herefore;
0 # #c
2
* "c
!
EcE1"
"c
!
EcE1" # #c
2
" # #c
2
" @ c
!
EcE1" -ote that c # 1.1L c
!
# 1.!1L c
2
# 1.221
" # 10001.221" @ 1.!1E1.1E1"
" # 1221 @ 2.21
Gounded to the nearest dollar& " # 40!.
%he correct answer is 7.
'. %he formula for calculating compound interest is / # #1 E rDn"
nt
where the
variables represent the following;
/ # amount of mone, accumulated after t ,ears principal E interest"
# # principal investment
r # interest rate annual"
n # number of times per ,ear interest is compounded
t # number of ,ears
3n this case& " represents the un(nown principal& r # '.& n # 4 since the compounding
is done 4uarterl,& and t # .5 since the time frame in 4uestion is half a ,ear 6 months".
Pou can solve this problem without using compound interest. '. interest over half a
,ear& however that interest is compounded& is approximatel, 4. interest. $o& to
compute the principal& it>s actuall, a ver, simple calculation;
100 # .04"
!500 # "
%he correct answer is C.
6.
%o solve a population growth 4uestion& we can use a population chart to trac( the
growth. %he annual growth rate in this 4uestion is un(nown& so we will represent it as
". For example& if the population doubles each ,ear& " # !L if it grows b, 50. each
,ear& " # 1.5. )ach ,ear the population is multiplied b, this factor of ".
%ime Oopulation
-ow 500
in 1 ,ear 500"
in ! ,ears 500"
!
; ;
in n ,ears 500"
n
%he 4uestion is as(ing us to find the minimum number of ,ears it will ta(e for the
herd to double in number. 3n other words& we need to find the minimum value of n
that would ,ield a population of 1000 or more.
5e can represent this as an ine4ualit,;
500"
n
V 1000
"
n
V !
3n other words& we need to find what integer value of n would cause "
n
to be greater
than !. %o solve this& we need to (now the value of ". %herefore& we can rephrase this
4uestion as; 95hat is "& the annual growth factor of the herdQ:
1" 3-$BFF373)-%; %his tells us that in ten ,ears the following ine4ualit, will hold;
500"
10
V 5000
"
10
V 10
%here are an infinite number of growth factors& "& that satisf, this ine4ualit,.
For example& " # 1.5 and " # ! both satisf, this ine4ualit,.
3f " # !& the herd of antelope doubles after one ,ear.
3f " # 1.5& the herd of antelope will be more than double after two ,ears 5001.5"1.5"
# 500!.!5".
!" $BFF373)-%; %his will allow us to find the growth factor of the herd. 5e can
represent the growth factor from the statement as y. -/%) y does not necessaril,
e4ual !" because " is a growth factor. For example& if the herd actuall, grows at a rate
of 10. each ,ear& " # 1.1& but y # 1.!& i.e. !0."
%ime Oopulation
-ow 500
in 1 ,ear 500y
in ! ,ears 500y
!
0ccording to the statement& 500y
!
# 6'0
y
!
# 6'0@500
y
!
# 46@!5
y # 7@5 /G 1.4 y canUt be negative because we (now the herd is growing"
%his means that the h,pothetical double rate from the statement represents an annual
growth rate of 40..
%he actual growth rate is therefore !0.& so " # 1.!.
%he correct answer is +.
10.
5e need two additional pieces of information to solve this problem& which can be
rephrased as D8ow fre4uentl, does the population double& and what is the population
size at an, given time immediatel, after it has doubledQD
1" 3-$BFF373)-%; 3f the population 4uadrupled during the last two hours& it
doubled twice during that interval& but this does not necessaril, mean that the
population doubled at 60 minute intervals. 3t ma, have& for example& doubled at 50 or
55 minute intervals. 5e cannot determine from statement 1" how fre4uentl, the
population is doubling.
!" 3-$BFF373)-%; %his statement does not give an, information about how
fre4uentl, the population is doubling.
1" 0-C !" $BFF373)-%; $tatement 1" indicates that the cells divided two hours
ago. 1et " e4ual the population immediatel, after that division. $tatement 1" also
indicates that
4" # " E 2&750
2" # 2&750
" # 1&!50
<iven that the population doubled to 1&!50 cells precisel, two hours ago and will
double to 40&000 cells precisel, three hours from now& we can determine how
fre4uentl, the population is doubling and therefore what the population will be four
hours from now.
5hile further calculation is unnecessar, at this point& it can be shown that
40&000@1&!50 # 2! # !
5
. %he population therefore doubles five times during that five
hour span between two hours ago and three hours hence" at one hour intervals. Four
hours from now& the population will double from 40&000 cells to '0&000 cells and will
then be destro,ed.
%he correct answer is 7.
!!.
3n order to answer this 4uestion& we need to (now the formula for compound interest;
$E is the future value.
# is the present value or the principle".
r is the rate of interest.
n is the number of compounding periods per ,ear.
t is the number of ,ears.
$ince <race deposited " dollars at a rate of & percent& compounded annuall,;
0nd since <eorgia deposited y dollars at a rate of & percent& compounded 4uarterl,
four times per ,ear";
$o the 4uestion becomes;
3s Q
$tatement 1 tells us that & # 4. %his tells us nothing about " or y. 3nsufficient.
$tatement ! tells us that 100y # &". %herefore& it must be true that y # &"@100. 5e can
use this information to simplif, the 4uestion;
%he 4uestion is now;
3s Q
5e (now from the 4uestion stem that & has a maximum value of 50. 3f we substitute
that maximum value for && we get;
$o the 4uestion is now;
3s Q
Bsing estimation& we can see that this ine4ualit, is true. $ince the maximum value of
& ma(es this ine4ualit, true& all smaller values of & will do so as well. %herefore& we
can answer D,esD to the rephrased 4uestion. $ufficient.
%he correct answer is +; $tatement ! alone is sufficient& but statement 1 alone is not.
!#.
3n order to answer this 4uestion& we need to recall the compound interest formula;
& where $E is the future value of the investment& #E is the present
value& r is the interest rate& and n is the number of compounding time periods.
3n this case& we do not (now the value of an, of the un(nowns and are as(ed to find
#E. 5e do& however& (now that the value of #E doubled. %herefore& $E # !#E. 5e
can use this to construct and simplif, the following e4uation;
%herefore& the interest rate . -ow we can loo( at the statements.
$tatement 1" tells us that the interest rate was between 26. and 45.. %herefore& the
value of is between 26 and 45.
3f n # 1& then . %his value is not between
26 and 45. %herefore& n does not e4ual 1.
3f n # !& then . -ote that the s4uare root
of ! is approximatel, 1.4." 40 is between 26 and 45& so ! is a possible value of n.
7an n be greater than !Q $ince the value of r 40" is almost at the lower limit of the
given range 26 to 45" when n # !& it is not possible that increasing the value of n to 2
resulting in our ta(ing the cube root of !& which is approximatel, 1.!6" would ,ield a
value of r that is above 26.
$o n must e4ual ! and r must be approximatel, 40. +ut this does not tell us the value
of #E.
$tatement !" tells us that the sale value of the bond would have been approximatel,
!&744 if the period of investment had been one month longer. 5e can set up the
following e4uation;
%his does not allow us to find a value for #E. $tatement !" is insufficient.
3f we ta(e the statements together& we can substitute the values of r and n derived
from statement 1";
%herefore& the approximate value of the original investment is X1&000.
%he correct answer is 7& both statements together are sufficient& but neither statement
alone is sufficient.
13.
Lets say:
I = the original amount of bacteria
F = the final amount of bacteria
t = the time bacteria grows
If the bacteria increase by a factor of x every y minutes, we can represent the growth of the bacteria with
the equation:
F = I(x)
t/y

o un!erstan! why, lets assign some values to I, " an! y:
I = #$$
x = %
y = &
If the bacteria start off #$$ in number an! they !ouble every & minutes, after & minutes there will be
#$$(%) bacteria' Let(s construct a table to trac) the growth of the bacteria:
t (time) F (final count)
& #$$(%) = #$$(%)
#
* #$$(%)(%) = #$$(%)
%
+ #$$(%)(%)(%) = #$$(%)&
#% #$$(%)(%)(%)(%) = #$$(%)
,
-e can generali.e the F values in the table as #$$(%)
n
'
he #$$ represents the initial count, I'
he % represents the factor of growth (in this problem x)'
he n represents the number of growth perio!s' he number of growth perio!s is foun! by !ivi!ing the
time, t, by the amount of time it ta)es to complete a perio!, y'
/rom this e"ample, we can e"trapolate the general formula for e"ponential growth: F = I(")
t/y
his question as)s us how long it will ta)e for the bacteria to grow to #$,$$$ times their original amount'
he bacteria will have grown to #$,$$$ times their original amount when F = #$,$$$I.
If we plug this into the general formula for e"ponential growth, we get: #$,$$$I= I(x)
t/y
or

#$,$$$ =
(x)
t/y
.
he question is as)ing us to solve for t'
(#) 01//I2I34: his statement tells us that x
#/y
=#$' If we plug this value into the equation we can
solve for t'
#$,$$$ = (x)
t/y
#$,$$$ =5(x)
1/y
6
t
#$,$$$ =(#$)
t
t = ,
(%) 01//I2I34: he bacteria grow one hun!re!fol! in % minutes, that is to say they grow by a factor of
#$
%
' 0ince e"ponential growth is characteri.e by a constant factor of growth (i'e' by " every y minutes),
for the bacteria to grow #$,$$$ fol! (i'e' a factor of #$
,
), they will nee! to grow another % minutes, for a
total of four minutes (#$
%
" #$
%
= #$
,
)'
he correct answer is 7, 3829 statement 8L:43 is sufficient to answer the question'
=+T>/S
1. First& let us rephrase the 4uestion. $ince we need to find the fraction that is at least
twice greater than 11@50& we are loo(ing for a fraction that is e4ual to or greater
than !!@50. Further& to facilitate our anal,sis& note that we can come up with an
eas, benchmar( value for this fraction b, doubling both the numerator and the
denominator and thus expressing it as a percent; !!@50 # 44@100 # 44.. %hus& we
can rephrase the 4uestion; 95hich of the following is greater than or e4ual to
44.Q:
-ow& letUs anal,ze each of the fractions in the answer choices using benchmar(
values;
!@5; %his fraction can be represented as 40.& which is less than 44..
11@24; %his value is slightl, less than 11@22 or 1@2. %herefore& it is smaller than
44..
42@66; -ote that the fraction 42@66 is smaller than 44@100& since fractions get
smaller if the same number in this case integer 1" is subtracted from both the
numerator and the denominator.
'@!1; 5e (now that '@!1 is a little less than '@!0 or !@5. %hus& '@!1 is less than
44..
6@!0; Finall,& note that b, multipl,ing the numerator and the denominator b, 5&
we can represent this fraction as 45@100& thus concluding that this fraction is
greater than 44..
%he correct answer is ).
!. 1etUs denote the number of Huniors and seniors at the beginning of the ,ear as (
and s& respectivel,. 0t the beginning of the ,ear& the ratio of Huniors to seniors was
2 to 4;
(@s # 2@4. %herefore& ( # 0.75s
0t the end of the ,ear& there were ( * 10" Huniors and s * !0" seniors.
0dditionall,& we (now that the ratio of Huniors to seniors at the end of the ,ear
was 4 to 5. %herefore& we can create the following e4uation;
1etUs solve this e4uation b, substituting ( # 0.75s;
( * 10" # 0.'s * !0"
0.75s * 10" # 0.'s * 16
0.'s * 0.75s # 16 * 10
0.05s # 6
s # 1!0
%hus& there were 1!0 seniors at the beginning of the ,ear.
%he correct answer is ).
2. For a fraction 4uestion that ma(es no reference to specific values& it is best to
assign a smart number as the Dwhole valueD in the problem. 3n this case we>ll use
20 since that is the least common denominator of all the fractions mentioned in
the problem.
3f there are 20 students in the class& 2@5 or 1'& left for the field trip. %his means that 1!
students were left behind.
1@2 of the 1! students who sta,ed behind& or 4 students& didn>t want to go on the field
trip.
%his means that ' of the 1! who sta,ed behind did want to go on the field trip.
5hen the second vehicle was located& half of these ' students or 4& were able to Hoin
the other 1' who had left alread,.
( * 10
s * !0
#
4
5
( * 10
s * !0
#
0.
'
%hat means that !! of the 20 students ended up going on the trip. !!@20 reduces to
11@15 so the correct answer is 7.
4. %he ratio of bo,s to girls in 7lass 0 is 2 to 4. 5e can represent this as an
e4uation; b@+ # 2@4. 5e can isolate the bo,s;
4b # 2+
b # 2@4"+
1et>s call the number of bo,s in 7lass + "& and the number of girls in 7lass + y.
5e (now that the number of bo,s in 7lass + is one less than the number of bo,s in
7lass 0. %herefore& " # b F 1. 5e also (now that the number of girls in 7lass + is
two less than the number of girls in 7lass 0. %herefore& y # + F !. 5e can substitute
these in the combined class e4uation;
%he combined class has a bo,@girl ratio of 17 to !!; b E ""@+ E y" # 17@!!.
b E b F 1"@+ E + F !" # 17@!!
!b F 1"@!+ F !" # 17@!!
7ross*multipl,ing ,ields;
44b F !! # 24+ F 24
$ince we (now that b # 2@4"+& we can replace the b;
442@4"+ F !! # 24+ F 24
22+ F !! # 24+ F 24
1! # +
0lternativel,& because the numbers in the ratios and the answer choices are so low&
we can tr, some real numbers. %he ratio of bo,s to girls in 7lass 0 is 2;4& so here are
some possible numbers of bo,s and girls in 7lass 0;
+;<
2;4
6;'
6;1!
%he ratio of bo,s to girls in 7lass + is 4;5& so here are some possible numbers of bo,s
and girls in 7lass 0;
+;<
4;5
';10
1!;15
5e were told that there is one more bo, in 7lass 0 than 7lass +& and two more girls
in 7lass 0 than 7lass +. 3f we loo( at our possibilities above& we see that this
information matches the case when we have 6 bo,s and 1! girls in 7lass 0 and '
bo,s and 10 girls in 7lass +. Further& we see we would have 6 E ' # 17 bo,s and 1!
E 10 # !! girls in a combined class& so we have the correct 17;!! ratio for a combined
class. 5e (now now there are 1! girls in 7lass 0.
%he correct answer is ).
5.
5e can solve this problem b, choosing a smart number to represent the size of the
bac( lawn. 3n this case& we want to choose a number that is a multiple of ! and 2 the
denominators of the fractions given in the problem". %his wa,& it will be eas, to split
the lawn into halves and thirds. 1et>s assume the size of the bac( lawn is 6.
size bac( lawn" # 6 units
size front lawn" # 1@2"size bac( lawn" # ! units
size total lawn" # size bac( lawn" E size front lawn" # ' units
-ow we can use these numbers to calculate how much of each lawn has been mowed;
Front lawn; 1@!"!" # 1 unit
+ac( lawn; !@2"6" # 4 unit
$o& in total& 5 units of lawn have been mowed. %his represents 5@' of the total&
meaning 2@' of the lawn is left unmowed.
0lternativel,& this problem can be solved using an algebraic approach. 1et>s assume
the size of the front lawn is " and size of the bac( lawn is y. $o& John has mowed
1@!"" and !@2"y& for a total of 1@!"" E !@2"y. 5e also (now that " # 1@2"y.
$ubstituting for " gives;
1@!"" E !@2"y
1@!"1@2"y E !@2"y
1@6"y E !@2"y
5@6"y # lawn mowed
%he total lawn is the sum of the front and bac(& " E y. 0gain& substituting for " gives
1@2"y E y # 4@2"y. $o& the fraction of the total lawn mowed is;
%he correct answer is 7.
6. 5e (now that the student to teacher ratio at the school is 16 to 1& and the total
number of people is 510. %herefore;
-umber of students # 16@17"510" # 4'0
-umber of teachers # 1@17"510" # 20
Mindergarten students ma(e up 1@5 of the student population& so;
-umber of (indergarten students # 1@5"4'0" # 66
lawn mowed
total lawn
5@6"y
4@2"y
#
5@6"
4@2"
# 5@6" A 2@4" # 15@!4 # 5@'. %his leaves 2@' unmowed.
Fifth and sixth graders account for 1@2 of the remainder after (indergarten
students are subtracted from the total"& therefore;
-umber of 5th and 6th grade students # 1@2"4'0 F 66" # 1@2"2'4" # 1!'
$tudents in first and second grades account for 1@4 of all the students& so;
-umber of 1st and !nd grade students # 1@4"4'0" # 1!0
$o far& we have accounted for ever, grade but the 2rd and 4th grades& so the,
must consist of the students left over;
-umber of 2rd and 4th grade students # %otal students F students in other grades
-umber of 2rd and 4th grade students # 4'0 F 66 F 1!' F 1!0 # 126
3f there are an e4ual number of students in the third and fourth grades& then;
-umber of 2rd grade students # 126@! # 6'
%he number of students in third grade is 6'& which is fewer than 66& the number of
students in (indergarten. %he number of students in 2rd grade is thus 66 F 6' # !'
fewer than the number of (indergarten students.
%he correct answer is 7.
7. 50 million can be represented in scientific notation as 5 x 10
7
. Gestating this
figure in scientific notation will enable us to simplif, the division re4uired to
solve the problem. 3f one out of ever, 5 x 10
7
stars is larger than the sun& we must
divide the total number of stars b, this figure to find the solution;
# 4@5 x 10
11*7"
# 0.' x 10
4

%he final step is to move the decimal point of 0.' four places to the right& with a result
of '&000.
%he correct answer is 7.
'. For a fraction word problem with no actual values for the total& it is best to plug
numbers to solve.
$ince 2@5 of the total cups sold were small and !@5 were large& we can arbitraril,
assign 5 as the number of cups sold.
%otal cups sold # 5
$mall cups sold # 2
4 x 10
11
5 x 10
7
1arge cups sold # !
$ince the large cups were sold at 7@6 as much per cup as the small cups& we (now;
#rice
lar+e
# 7@6"#rice
small
1et>s assign a price of 6 cents per cup to the small cup.
Orice of small cup # 6 cents
Orice of large cup # 7 cents
-ow we can calculate revenue per cup t,pe;
1arge cup sales # 4uantit, A cost # ! A 7 # 14 cents
$mall cup sales # 4uantit, A cost # 2 A 6 # 1' cents
%otal sales # 2! cents
%he fraction of total revenue from large cup sales # 14@2! # 7@16.
%he correct answer is 0.
6. %his problem can be solved most easil, b, pic(ing smart numbers and assigning
values to the portion of each ingredient in the dressing. 0 smart number in this
case would be one that enables ,ou to add and subtract ingredients without having
to deal with fractions or decimals. 3n a fraction problem& the Ysmart numberU is
t,picall, based on the least common denominator among the given fractions.
%he two fractions given& 5@' and 1@4& have a least common denominator of '.
8owever& we must also consider the e4ual parts salt& pepper and sugar.
+ecause 1@4 # !@'& the total proportion of oil and vinegar combined is 5@' E !@' #
7@'. %he remaining 1@' of the recipe is split three wa,s; 1@!4 each of salt& pepper&
and sugar. !4 is therefore our least common denominator& suggesting that we
should regard the salad dressing as consisting of !4 units. 1etUs call them cups for
simplicit,& but an, unit of measure would do. 3f properl, mixed& the dressing
would consist of
5@' A !4 # 15 cups of olive oil
1@4 A !4 # 6 cups of vinegar
1@!4 A !4 # 1 cup of salt
1@!4 A !4 # 1 cup of sugar
1@!4 A !4 # 1 cup of pepper
?iguel accidentall, doubled the vinegar and omitted the sugar. %he composition
of his bad salad dressing would therefore be
15 cups of olive oil
1! cups of vinegar
1 cup of salt
1 cup of pepper
%he total number of cups in the bad dressing e4uals !6. /live oil comprises 15@!6
of the final mix.
%he correct answer is 0.
10.
%his problem never tells us how man, boo(s there are in an, of the libraries. 5e can&
therefore& pic( numbers to represent the 4uantities in this problem. 3t is a good idea to
pic( $mart -umbers& i.e. numbers that are multiples of the common denominator of
the fractions given in the problem.
3n this problem& 8arold brings 1@2 of his boo(s while ?illicent brings 1@!. %he
denominators& ! and 2& multipl, to 6& so let>s set 8arold>s librar, capacit, to 6 boo(s.
%he problem tells us ?illicent has twice as man, boo(s& so her librar, capacit, is 1!
boo(s. 5e use these numbers to calculate the size of the new home>s librar, capacit,.
1@2 of 8arold>s 6*boo( librar, e4uals ! boo(s. 1@! of ?illicent>s 1!*boo( librar,
e4uals 6 boo(s. %ogether& the, bring a combined ' boo(s to fill their new librar,.
%he fraction we are as(ed for& new home>s librar, capacit," @ ?illicent>s old librar,
capacit,"& therefore& is '@1!& which simplifies to !@2.
%he correct answer is +.
11.
%he ratio of dogs to cats to bunnies Cogs; 7ats; +unnies" can be expressed as 2x;
5x ; 7x. 8ere& x represents an Dun(nown multiplier.D 3n order to solve the problem&
we must determine the value of the un(nown multiplier.
7ats E +unnies # 4'
5x E 7x # 4'
1!x # 4'
x # 4
-ow that we (now that the value of x the un(nown multiplier" is 4& we can
determine the number of dogs.
Cogs # 2x # 24" # 1!
%he correct answer is 0.
1!. +o,s # !n@5& girls # 2n@5
<irls stud,ing <erman # all girls" F girls stud,ing $panish" F girls stud,ing
French"
%he correct answer is ).
12.
$ince the problem deals with fractions& it would be best to pic( a smart number to
represent the number of ball pla,ers. %he 4uestion involves thirds& so the number we
pic( should be divisible b, 2. 1et>s sa, that we have 6 right*handed pla,ers and 6 left*
handed pla,ers remember& the 4uestion states that there are e4ual numbers of righties
and lefties".
2n
<irls stud,ing $panish # A
5
1

2
n
#
5
2n
<irls not stud,ing $panish #
F
5
n

5
!n
#
5
!n
<irls stud,ing French # A
5
5

6
n
#
2
2n
<irls stud,ing <erman # F
5
n

5
n
F
2
!n
<irls stud,ing <erman # F
5
n

2
n
#
15
%wo*thirds of the pla,ers are absent from practice& so that is !@2"1'" # 1!. %his
leaves 6 pla,ers at practice. /f these 6 pla,ers& one*third were left*handed. %his
,ields 1@2"6" # ! left*handed pla,ers at practice and 6 F ! # 7 left*handed pla,ers
-/% at practice. $ince ! of the 6 pla,ers at practice are lefties& 6 F ! # 4 pla,ers at
practice must be righties& leaving 6 F 4 # 5 righties -/% at practice.
%he 4uestion as(s us for the ratio of the number of righties not at practice to the
number of lefties not at practice. %his must be 5 ; 7 or 5@7.
%he correct answer is 7.
14.
5e are told that bag + contains red and white marbles in the ration 1;4. %his implies
that 5
+
& the number of white marbles in bag +& must be a multiple of 4.
5hat can we sa, about 5
0
& the number of white marbles in bag 0Q 5e are given two
ratios involving the white marbles in bag 0. %he fact that the ratio of red to white
marbles in bag 0 is 1;2 implies that 5
0
must be a multiple of 2. %he fact that the ratio
of white to blue marbles in bag 0 is !;2 implies that 5
0
must be a multiple of !.
$ince 5
0
is both a multiple of ! and a multiple of 2& it must be a multiple of 6.
5e are told that 5
0
E 5
+
# 20. 5e have alread, figured out that 5
0
must be a
multiple of 6 and that 5
+
must be a multiple of 4. $o all we need to do now is to test
each candidate value of 5
0
i.e. 6& 1!& 1'& and !4" to see whether& when plugged into
5
0
E 5
+
# 20& it ,ields a value for 5
+
that is a multiple of 4. 3t turns out that 5
0
# 6
and 5
0
# 1' are the onl, values that meet this criterion.
Gecall that the ratio of red to white marbles in bag 0 is 1;2. 3f there are 6 white
marbles in bag 0& there are ! red marbles. 3f there are 1' white marbles in bag 0&
there are 6 red marbles. %hus& the number of red marbles in bag 0 is either ! or 6.
/nl, one answer choice matches either of these numbers.
%he correct answer is C.
15. 3nitiall, the ratio of +; 7; ) can be written as '"; 5"; 2". Gecall that ratios
alwa,s emplo, a common multiplier to calculate the actual numbers."
0fter removing 4 pounds of clothing& the ratio of boo(s to clothes is doubled. %o
double a ratio& we double Hust the first numberL in this case& doubling ' to 5 ,ields a
new ratio of 16 to 5. %his can be expressed as follows;
7ross multipl, to solve for ";
40" # '0" F 64
40" # 64
" # '@5
%he 4uestion as(s for the approximate weight of the electronics in the suitcase. $ince
there are 2" pounds of electronics there are 2 A '@5" # !4@5 or approximatel, 5
pounds of electronics in the suitcase.
boo(s
clothing
#
'"
5" F 4
#
16
5
%he correct answer is 7.
16.
3t is useful to thin( of the ratio as 1" ; !" ; 4"& where " is the Dmissing multiplierD that
,ou use to find the actual numbers involved. For example& if " # 1& then the numbers
of hours wor(ed b, the three men are 1& !& and 4. 3f " # !& then the numbers are !& 4&
and '. 3f " # 11& then the numbers are 11& !!& and 44. -otice that these numbers all
retain the original ratio. 3f we (new the multiplier& we could figure out the number of
hours an, of the men wor(ed. $o we can rephrase the 4uestion as& D5hat is the
missing multiplierQD
$BFF373)-%; $ince the three men wor(ed a total of 46 hours and since 1" E
!" E 4" # 7"& we (now that 7" # 46. %herefore& " # 7. $ince +ob wor(ed !"
hours& we (now he wor(ed !7" # 14 hours.
$BFF373)-%; %his statement tells us that 4" # 1" E !1. %herefore& 2" # !1
and " # 7. $ince +ob wor(ed !" hours& we (now he wor(ed !7" # 14 hours.
%he correct answer is C.
17.
%he 4uestion as(s us to find the ratio of gross revenue of computers to printers& given
that the price of a computer is five times the price of a printer. 5e will prove that the
statements are insufficient either singl, or together b, finding two examples that
satisf, all the criteria but give two different ratios for the gross revenue of computers
to printers.
1" 3-$BFF373)-%; $tatement 1" sa,s that the ratio of computers to printers sold in
the first half of !002 was in the ratio of 2 to !& so let>s assume the, sold 2 computers
and ! printers. Bsing an example price of X5 and X1 indicates that the computer gross
was X15 and the printer gross was X!.
Curing the second half of !002& the ratio of computers to printers sold was ! to 1. For
example& the, ma, have sold ! computers and 1 printer grossing X10 and X1
respectivel,. 0dding in the first half revenue& we can calculate that the, would have
grossed X!5 and X2 respectivel, for the full ,ear.
0lternativel, for the second half of !002 the, ma, have sold 4 computers and !
printers& which is still in the ratio of ! to 1. 3n this case the, would have grossed X!0
and X! respectivel,. -ow adding in the first half revenue indicates the, would have
grossed X25 and X4 respectivel, for the full ,ear& which is a different ratio. %herefore
statement 1" is insufficient to give us a definitive answer.
!" 3-$BFF373)-%; $tatement !" tells us that a computer costs X1&000& but it tells us
nothing about the ratio or numbers of computers or printers sold.
1" and !" 3-$BFF373)-%; $tatement !" fixes the price of a computer at X1000& but
the counterexample given in the explanation of statement 1" still holds& so statements
1" and !" together are still insufficient.
%he correct answer is ).
1'. 1et " represent the amount of water in Oool K& and y represent the amount of
water in Oool P. 3f we let & represent the proportion of Oool P>s current volume
that needs to be transferred to Oool K& we can set up the following e4uation and
solve for &;
water currentl, in Oool K" E water transferred" # water currentl, in Oool P" F
water transferred"
" E &y # y F zy
" E !&y # y
!&y # y F "
$o& the value of z depends onl, on the ratio of the water currentl, in Oool K to the
water currentl, in
Gemember that " and y do -/% represent the capacities of either pool& but rather the
07%B01 0?/B-%$ of water in each pool.
1" $BFF373)-%; if we let K represent the capacit, of Oool K& then the amount of
water in Oool K is !@7"K. $o& " # !@7"K. 5e can calculate the total amount of water
in Oool P& or y& as follows; y # 6@7"K F !@7"K # 4@7"K. 5e can see that Oool P has
twice as much water
!" 3-$BFF373)-%; %his gives no information about the amount of water in Oool P.
%he correct answer is 0.
16. 5e can rewrite the information in the 4uestion as an e4uation representing the %&
the total dollar value of the sale;
1 E ? E $ # %
& #
y
!y
F
"
!y
& #
1
!

1
F
"
y
"
Oool P& or;
"
y
. %he rephrased 4uestion is; D5hat is
"
y
QD
as Oool K& or !" # y& or

"
y
# 1@!.
1 # the dollar amount received b, the partner with the largest share
? # the dollar amount received b, the partner with the middle second largest" share
$ # the dollar amount received b, the partner with the smallest share
5e are also told in the 4uestion that 1 # 5@'"%. %hus we can rewrite the e4uation as
follows;
5@'"% E ? E $ # %.
$ince the 4uestion as(s us the value of $& we can simplif, the e4uation again as
follows;
$ # ? E 2@'"%
%hus& in order to solve for $& we will need to determine the value of both ? and %.
%he 4uestion can be rephrased as& what is the value of ? E 2@'"%Q
-/% $BFF373)-%; %he first statement tells us that $ # 1@5"?. %his gives us
no information about % so statement one alone is not sufficient.
$BFF373)-%; %he second statement tells us that ? # 1@!"1 # X1 million.
0dditionall,& since we (now from the 4uestion that 1 # 5@'"%& then ? must
be e4ual to 1@! of 5@'%" or 5@16%". 5e can therefore solve for % as follows;
X2&!00&000 # %
5e can now easil, solve for $;
1 E ? E $ # %
! million E 1 million E $ # X2.! million
$ # .! million
%he correct answer is +.
!0. %he 4uestion as(s us to solve for the ratio of pennies p" to dimes d".
3-$BFF373)-%; %his tells us that the ratio of nic(els n" to dimes d" is 2;!.
%his gives us no information about the ratio of pennies to dimes.
3-$BFF373)-%; %his tells us that there is X7& or 700 cents in the pigg, ban(.
5e can write an e4uation for this as follows& using the value of each t,pe of
coin; 10d E 5n E p # 700. %his is not enough information for us to figure out
the ratio of p to d.
1" 0-C !" 3-$BFF373)-%; %a(en together& both statements still do not
provide enough information for us to figure out the ratio of p to d. For
example& there ma, be 2 nic(els& ! dimes& and 665 pennies in the pigg, ban(
this (eeps the ratio of nic(els to dimes at 2;! and totals to X7". 0lternativel,&
there ma, be 20 nic(els& !0 dimes& and 250 pennies this also (eeps the ratio
of nic(els to dimes at 2;! and totals to X7". 3n these ! cases the ratio of pennies
to dimes is not the same.
? # X1&000&000 #
5
16
%
%he correct answer is ).
!1. %o determine the ratio of 7hemical 0 to 7hemical 7& we need to find the amount
of each in the solution. %he 4uestion stem alread, tells us that there are 10
milliliters of 7hemical 7 in the final solution. 5e also (now that the original
solution consists of onl, 7hemicals 0 and + in the ratio of 2 to 7. %hus& we
simpl, need the original volume of the solution to determine the amount of
7hemical 0 contained in it.
$BFF373)-%; %his tells us that original solution was 50 milliliters. %hus&
there must have been 15 milliliters of 7hemical 0 to 25 milliliters of
7hemical +". %he ratio of 0 to 7 is 15 to 10 or 2 to !".
$BFF373)-%; %his tells us that the final solution was 60 milliliters. 5e (now
that this includes 10 milliliters of 7hemical 7. %his means the original
solution contained 50 milliliters. %hus& there must have been 15 milliliters of
7hemical 0 to 25 milliliter of 7hemical +". %he ratio of 0 to 7 is 15 to 10
or 2 to !".
%he correct answer is C.
!!. <iven woman; children#5;!
1". children; man#5;11& ,ou agree it is insufficient
!". 5\20& ,ou also agree it alone is insufficient
%ogether& w;c;m # !5;10;!! all have to be integers=" thus w#!5 and m#!!.
0nswer is 7
!2.
1et Kf@Pf is the full time in Civision K@P& and Kp@Pp is part time in K@P& K&P& and I
are number of emplo,ees in K& P& and I.
K#KfEKp
P#PfEPp
I#KEP
For 1& Pf@Pp\If@Ip& as a compensation& Kf@Kp should be greater than If@Ip
For!& ?ore than QIf @less than Qof the Ip should be greater than If@Ip
0nswer is C
#4.
%he total number of pigs and cows is 40.
For1& 7V!O
For !& OV1!
7ombine 1 and !& if O#12& 7 is 14Lif O#14& 7 is 1!& it is impossible.
0nswer is 7
!5. Oremise; one serving includes a certain number of dishes.we don>t (now the
exact number"&and a dish re4uires 2@! cups of pasta. it means 4P#mK& and
K#2@!pasta. "
Nuestion; nP re4uire how man, cups of pastaQ
1". if ?ali( ma(e K servings next time. 8e did prepare !K dishes last time.
!". ?ali( used 6 cups of pasta the last time he prepared this dish.it means !K#6".
3n this case& either condition one or condition two cannot deduce the final answer
in that the decisive factors m& n are un(nown.
0s a result& the correct answer is 7.

You might also like